Inference

Preview this deck

Which of the following best completes the passage below?

Adding ethanol to the gasoline used in cars reduces exhaust emissions while slightly increasing evaporation from gasoline tanks in cars and service stations. These evaporative emissions are a major component of the smog found in many large cities in the hot summer months but, except in hot weather, evaporative emissions pose less of a pollution problem than exhaust emissions do. Therefore, if air pollution were the only consideration, one could conclude that ________.


(A) ethanol should be added only to gasoline intended for use in large cities
(B) the benefits of using gasoline to which ethanol has been added are greater in hot weather than in cold weather
(C) the disadvantages of adding ethanol to gasoline are likely to be outweighed by the advantages, at least in cold weather
(D) it is not necessary to work at reducing exhaust emissions in large cities
(E) adding ethanol to the gasoline used in large cities will improve air quality in the cities in hot weather

Front

Star 0%
Star 0%
Star 0%
Star 0%
Star 0%

0.0

0 reviews

5
0
4
0
3
0
2
0
1
0

Active users

1

All-time users

1

Favorites

0

Last updated

3 years ago

Date created

Oct 5, 2020

Cards (104)

New

(0 cards)

Inference

(15 cards)

Which of the following best completes the passage below?

Adding ethanol to the gasoline used in cars reduces exhaust emissions while slightly increasing evaporation from gasoline tanks in cars and service stations. These evaporative emissions are a major component of the smog found in many large cities in the hot summer months but, except in hot weather, evaporative emissions pose less of a pollution problem than exhaust emissions do. Therefore, if air pollution were the only consideration, one could conclude that ________.


(A) ethanol should be added only to gasoline intended for use in large cities
(B) the benefits of using gasoline to which ethanol has been added are greater in hot weather than in cold weather
(C) the disadvantages of adding ethanol to gasoline are likely to be outweighed by the advantages, at least in cold weather
(D) it is not necessary to work at reducing exhaust emissions in large cities
(E) adding ethanol to the gasoline used in large cities will improve air quality in the cities in hot weather

Front

 

1. Adding ethanol to the gasoline -> giảm exhaust emissions nhưng tăng evaporation from gasoline tanks

2. Evaporation from gasoline tanks - thành phần chủ yếu dẫn đến  smog in many large cities in the hot summer

3. (Except in hot weather = Loại trừ mùa nóng) <=> vào mùa lạnh, evaporative emissions sẽ ÍT là vấn đề ô nhiễm hơn exhaust emissions

=> Adding ethanol to the gasoline vào Mùa lạnh thì  tốt hơn là vào Mùa nóng

 

Therefore, if air pollution were the only consideration, one could conclude that ___________.

C. the disadvantages of adding ethanol to gasoline are likely to be outweighed by the advantages, at least in cold weather

<-> Bất lợi của việc adding ethanol sẽ bị outweighed bởi "Có lợi", ít nhất là vào mùa đông

Back
Front

So.... -> conclusion (≈ inference)

Premise 1: Có khả năng một cách lý thuyết là bacteria developed on Mars và được đưa đến Trái Đất bởi meteorite

Premise 2: Các loài bacteria phát triển ở các planets khác nhau thì sẽ có thể có cấu trúc protein khác nhau rất lớn

Premise 3: Không có 2 bacteria nào ở Trái Đất có khác nhau đủ lớn để mà có nguồn gốc từ các hành tinh khác nhau

 

=> Khả năng cao là tất cả các bacteria có cùng một nguồn gốc (từ Mars) -> D.

 

B. Opposite

E. No info

Back

Finding of a survey of Systems magazine subscribers: Thirty percent of all merchandise orders placed by subscribers in response to advertisements in the magazine last year were placed by subscribers under age thirty-five.

Finding of a survey of advertisers in Systems magazine: Most of the merchandise orders placed in response to advertisements in Systems last year were placed by people under age thirty-five.

For both of the findings to be accurate, which of the following must be true?

A. More subscribers to Systems who have never ordered merchandise in response to advertisements in the magazine are age thirty-five or over than are under age thirty-five.

B. Among subscribers to Systems, the proportion who are under age thirty-five was considerably lower last year than it is now.

C. Most merchandise orders placed in response to advertisements in Systems last year were placed by Systems subscribers over age thirty-five.

D. Last year, the average dollar amount of merchandise orders placed was less for subscribers under age thirtyfive than for those age thirty-five or over.

E. Last year many people who placed orders for merchandise in response to advertisements in Systems were not subscribers to the magazine.

Front

Fact 1. Subcribers:  30% orders_ by res ads của những người dưới 35

=> sub<35: có ít hơn 30% Toàn bộ orders_res ads

Fact 2: Most (>50%) orders_res ads = people <35

=> Weighted average: Một số người đặt orders_res ads KHÔNG PHẢI sub (E)

 

C. Không thể so sánh được được % order đặt bởi Sub >35 trong tổng số đơn hàng res_ads

Back

Among people experience migraine headaches, some experience what doctors call "common" migraines, whereas others experience "classical" migraines. Siblings and spouses of common migraine sufferers are themselves twice as likely as the general population to experience common migraines. Siblings of classical migraine sufferers are four times more likely than the general population to experience classical migraines, whereas spouses of classical migraine sufferers are no more likely than the general population to experience such headaches.

The information above provides the most support for which of the following hypotheses?


(A) Susceptibility to classical migraines is more dependent on hereditary factors than is susceptibility to common migraines.

(B) Unmarried adults are more likely to suffer from classical migraines than they are to suffer from common migraines.

(C) People who do not experience migraine headaches are unlikely to have spouses who are migraine headache sufferers.

(D) Children of people who suffer from common migraines are more likely than the general population to experience a common migraine.

(E) Between one-quarter and one-half of the general population suffer from either common or classical migraine headaches.
 

Front

1. Common Migraine sufferers: Siblings and Spouses TWICE as likely to also have.

2. Classical migraine sufferers: Siblings FOUR TIMES as likely to also have. Spouses-no difference.

* Siblings <=> hereditary factors

=> A. đúng

 

B.  Unmarried adults: Ko có nhắc đến, ko được hiểu nhóm này là Siblings

C. <=> If (những người có spouses là migraine suffers), thì có (nhiều khả năng có migraine headaches) hơn -> không chính xác
-> Cần phải xét riêng từng loại migraine headaches: classical migraine thì dù có spouse bị bệnh cũng ko tăng khả năng partner còn lại bị classical migraine

-> Chỉ đúng với trường hợp common migraine

D. Children of people who suffer from common migraines: Ko có nhắc đến

E. Ko có gì để tính tỷ lệ dân số ở đây.

To prove this, you would have to make assumptions about not only the % of the population affected by migraines - but also the % of the population that is married or has a sibling.

Back

Junior biomedical researchers have long assumed that their hirings and promotions depend significantly on the amount of their published work. People responsible for making hiring and promotion decisions in the biomedical research field, however, are influenced much more by the overall impact that a candidate's scientific publications have on his or her field than by the number of those publications.

 

The information above, if accurate, argues most strongly against which of the following claims?

 

(A) Even biomedical researchers who are just beginning their careers are expected already to have published articles of major significance to the field.

 

(B) Contributions to the field of biomedical research are generally considered to be significant only if the work is published.

 

(C) The potential scientific importance of not-yet-published work is sometimes taken into account in decisions regarding the hiring or promotion of biomedical researchers.

 

(D) People responsible for hiring or promoting biomedical researchers can reasonably be expected to make a fair assessment of the overall impact of a candidate's publications on his or her field.

 

(E) Biomedical researchers can substantially increase their chances of promotion by fragmenting their research findings so that they are published in several journals instead of one.

Front

 

Argument:

- Juinor assumed - càng có nhiều published works thì càng có cơ hội được tuyển và tăng chức

- Tuy nhiên, người quyết định tăng chức: công trình có ảnh hưởng chung tới lĩnh vực thì sẽ thuyết phục hơn là số lượng

Question stem: thông tin bên trên có weaken thông tin bên dưới hay không

=> Cần phải liên quan đến cơ hội thăng chức, tuyển dụng  VÀ số lượng bài post/ chất lượng bài post

 

A. Những researcher mới bắt đầu công việc sẽ ĐƯỢC kì vọng là sẽ publish articles of major significance to the field -> argument ko đánh giá về kì vọng

B. Đóng góp vẫn chỉ có thể quan trọng nếu công trình được published-> argument không đánh giá chất lượng bài nghiên cứu

C. sometimes -> luôn luôn đúng

D. Người quyết định đưa ra đánh giá công bằng

-> supported

E. Researcher CÓ thể tăng cơ hội bằng cách chia nhỏ findings -> Weakened

Back

After observing thirty-year-old albatrosses, biologists determined that migration distance in mature albatrosses is directly tied to the lifespan of other fledglings in their brood. Albatrosses that hatched alone or whose fellow fledglings did not survive after their first molting were observed to follow shorter migration paths after thirty years than those that had been accompanied by one or more other fledglings.

If the biologists' determination is accurate, it follows that

A. a few albatrosses who molted before maturity are capable of migrating further after thirty years as a result

B. the migration distance of thirty-year-old albatrosses is connected to the migration distance of other fledglings

C. it is the survival of other fledglings rather than just the birth of those fledglings that more directly affects the migration distance of albatrosses after thirty years

D. albatrosses that migrated long distances after their first molting are more likely to migrate long distances after thirty years as well

E. the lifespan of other fledglings is more important to albatrosses after their first molting than it is after thirty years

Front

1. QUÃNG ĐƯỜNG DI CƯ của mature albatrosses (>=30 tuổi) phụ thuộc vào TUỔI THỌ-LIFESPAN of other fledglings

2. Thirty-year-old albatrosses - nếu ở 1 mình khi nở trứng/ bạn bè bị chết khi thay lông - sẽ di cư với cung đường NGẮN HƠN - Thirty-year-old albatrosses accompanied by one or more other fledglings.

-> the findings follow that:

 

A. Kết quả là: Một số ít albatrosses who molted before maturity - có khả năng di cư xa hơn sau 30 năm

-> Ko nhắc đến những con albatrosses mà thay lông trước khi maturity

B. Khoảng cách di cư của thirty-year-old albatrosses - liên kết với - migration distance of other fledglings

-> Khoảng cách di cư của thirty-year-old albatrosses chỉ liên quan đến LIFESPAN/ THE SURVIVAL of other fledglings

 

D. Không có thông tin nào liên quan đến connection giữa  "Khoảng cách di cư sau khi thay lông lần đầu" và "Khoảng cách di cư sau 30 tuổi" của albatrosses

 

E. Tuổi thọ của những con non khác - đối với albatrosses - quan trọng HƠN tại thời điểm sau khi chúng thay lông VS. sau 30 năm

-> Quan trọng là như thế nào?

 

C. Sự sống của những con non, thay vì sự sinh ra của chúng, ảnh hưởng trực tiếp HƠN đến quãng đường di cư của albatrosses sau 30 năm

Đúng. Vì albatross KO có fellows KO THỂ ĐI XA. Nhưng có fellow mà FELLOW chết ngay khi thay lông - cũng ko thể đi xa

Back

Critic: Emily Dickinson's poetry demonstrates that meaning cannot reside entirely within a poem itself, but is always the unique result of an interaction between a reader's system of beliefs and the poem; and, of course, any two readers from different cultures or eras have radically different systems of beliefs.

If the critic's statements are true, each of the following could be true EXCEPT:

A. A reader's interpretation of a poem by Dickinson is affected by someone else's interpretation of it.
B. A modern reader and a nineteenth-century reader interpret one of Shakespeare's sonnets in the same way.
C. A reader's interpretation of a poem evolves over time.
D. Two readers from the same era arrive at different interpretations of the same poem.
E. A reader's enjoyment of a poem is enhanced by knowing the poet's interpretation of it.

Front

Tuy là nói thơ ca của Emily Dickinson, nhưng các bài thơ ở đây không chỉ giới hạn vào thơ của Emily Dickinson

Do argument dùng từ "a poem itself" - 1 bài thơ chung chung

B. Không thể xảy ra trường hợp "1 người hiện tại và 1 người ở thế kỉ 19 cùng đọc hiểu thơ của Shakespeare theo cùng 1 cách"

 

A. C. E. có thể xảy ra, nó không được nhắc đến cụ thể trong argument

D. Hoàn toàn có thể xảy ra với 2 readers ở 2 nền văn hoá khác nhau

 

Back

The symptoms that US president Zachary Taylor began showing five days before his death are consistent with arsenic poisoning. Recent examinations of Taylor's bones, however, revealed levels of arsenic comparable to those found in the bones of Taylor's contemporaries. These levels are much lower than the levels of arsenic that remain in the bones of arsenic poisoning victims who live for more than a day after ingesting a lethal does of the poison.

Which of the following is most strongly supported by the statements given?

(A) The symptoms that Taylor began showing five days before his death are consistent with poisoning other than arsenic poisoning

(B) Taylor's death was not the result of arsenic poisoning

(C) The symptoms that Taylor began showing five days before his death were not caused by a lethal dose of arsenic

(D) The symptoms that Taylor began showing five days before his death grew more severe each day

(E) It is unusual for a person who has ingested a lethal dose of arsenic to live for more than a day

Front

Fact 1. The symptoms five days before his death were consistent with arsenic poisoning.

Fact 2. Mức độ arsenic trong xương của ông ấy thấp hơn so với mức độ arsenic trong xương của những người uống 1 liều arsenic đủ để chết (a lethal does of the poison.)

=> Khả năng cao là ông này không chết vì 1 liều arsenic đủ để chết ngay. (ví dụ. Có thể là ông ấy chết bởi vì bị hạ độc arsenic lượng nhỏ từ từ)

 

A. >< Fact 1

B. Chỉ có thể infer là ông này không chết vì 1 liều arsenic đủ để chết ngay. Có thể là ông ấy chết bởi vì bị hạ độc arsenic lượng nhỏ từ từ)

C. Đúng. Liên quan đến cả fact 1 và fact 2

-> Triệu chứng của ông ấy bắt đầu trước khi chết 5 ngày không bị gây ra bởi  1 liều arsenic đủ để chết ngay.

D. servere: Ko  có thông tin liên quan

E. Unusual: Ko có thông tin liên quan

Back

Crimes are mainly committed by the young, and for this reason merely increasing the number of police officers or expenditures on police services has little effect on reducing the crime rate. In fact, the only factor associated with a crime-rate drop is a decrease in the number of people in the community aged fourteen to thirty.

The findings above can best serve as part of an argument against

(A) the likelihood that any law enforcement program will be effective in reducing the crime rate within a short time
(B) increasing prison terms for young people found guilty of crimes
(C) introducing compulsory military conscription for people aged seventeen to nineteen
(D) raising the age at which students are permitted to leave school
(E) a community’s plan to increase the number of recreational and educational activities in which young adults can participate

Front

1. Tăng số lượng cảnh sát/ chi phí dịch vụ cảnh sát: có rất ít tác dụng giảm tỷ lệ tội phạm

2. Yếu tố duy nhất liên quan đến việc giảm tỷ lệ tội phạm là giảm số lượng người từ 14-30 tuổi (nhàn rỗi, tự do trong cộng đồng)

=> The finding weaken ...

B. Support. Tăng thời gian ngồi tù cho người trẻ phạm tội -> giảm số lượng người trẻ

C. Support. Áp dụng ctrinh quân sự bắt buộc -> giảm số lượng người trẻ

D. Support. Tăng độ tuổi mà lúc đó học sinh được phép rời khỏi trường học -> giảm số lượng người trẻ (nhàn rỗi, tự do trong cộng đồng)

E. Support. Thêm các hoạt động giáo dục và giải trí để người trẻ có thể tham gia -> giảm số lượng người trẻ (nhàn rỗi, tự do trong cộng đồng)

A. Weaken: Khả năng thành công của các chương trình thực thi luật pháp -> ko giúp  giảm số lượng người trẻ nhàn rỗi

Back

Canadians now increasingly engage in “out-shopping,” which is shopping across the national border, where prices are lower. Prices are lower outside of Canada in large part because the goods-and-services tax that pays for Canadian social services is not applied.

Which one of the following is best supported on the basis of the information above?


(A) If the upward trend in out-shopping continues at a significant level and the amounts paid by the government for Canadian social services are maintained, the Canadian goods-and-services tax will be assessed at a higher rate.

(B) If Canada imposes a substantial tariff on the goods bought across the border, a reciprocal tariff on cross-border shopping in the other direction will be imposed, thereby harming Canadian businesses.

(C) The amounts the Canadian government pays out to those who provide social services to Canadians are increasing.

(D) The same brands of goods are available to Canadian shoppers across the border as are available in Canada.

(E) Out-shopping purchases are subject to Canadian taxes when the purchaser crosses the border to bring them into Canada

Front

 

 

Canadian social services được trả thông qua  the goods-and-services tax (GST)

GST= sales * tax rate

Khi mà Canadians mua hàng nhiều ở ngoài biên giới ->Sales hàng hoá hội địa giảm

=> để giữ nguyên lượng tiền trả cho Canadian social services -> tax rate phải tăng

Back
Front

 

1. Left-handed persons: có khả năng mắc bệnh miễn dịch (ví dụ: dị ứng) THƯỜNG XUYÊN hơn Right-handed persons

2. Left-handed persons: Não phải tốt hơn -> Lý luận toán học tốt hơn

=> Tỷ lệ người thuận tay trái trong số những người giỏi Toán - CAO HƠN - Tỷ lệ người thuận tay trái trong số những người kém Toán (C)

-> support what?

 

A. (1) If X then Y - ko có nghĩa là (A): Đa số người mắc di ứng - là người thuận tay trái: If Y,  then X

G/s 90% dân số là thuận tay phải, thì kết luận này có thể sai.

B. Đa số người thuận tay trái, KO PHẢI NHÀ TOÁN HỌC THUẬN TAY TRÁI, mắc dị ứng

D. Allergy - ko có liên quan trực tiếp gì đến - mathematics

E. Tỷ lệ người mắc bệnh miễn dịch/ TỔNG DÂN SỐ - CAO HƠN Tỷ lệ thuận tay trái hoặc kém Toán/ TỔNG DÂN SỐ????

C. Người tay trái thường giỏi Toán => tỷ lệ người tay trái trong nhóm giỏi toán sẽ cao hơn tỷ lệ người tay trái trong nhóm không giỏi toán

Back

Between 1975 and 1985, nursing-home occupancy rates averaged 87 percent of capacity, while admission rates remained constant, at an average of 95 admissions per 1,000 beds per year. Between 1985 and 1988, however, occupancy rates rose to an average of 92 percent of capacity, while admission rates declined to 81 per 1,000 beds per year.

If the statements above are true, which of the following conclusions can be most properly drawn?

(A) The average length of time nursing-home residents stayed in nursing homes increased between 1985 and 1988.

(B) The proportion of older people living in nursing homes was greater in 1988 than in 1975.

(C) Nursing home admission rates tend to decline whenever occupancy rates rise.

(D) Nursing homes built prior to 1985 generally had fewer beds than did nursing homes built between 1985 and 1988.

(E) The more beds a nursing home has, the higher its occupancy rate is likely to be.

Front

1. nursing-home occupancy rates: số lượng người trung bình/ 100 giường bệnh tại 1 thời điểm (87% (1975 - 1985) -> 92/1000 (1985 - 1988))

2. admission rates. <=> Số lượng người khác nhau được nhập THÊM vào viện trong 1 năm (95/1000 (1975 - 1985) -> 81/1000 (1985 - 1988))

<=> Giống như kiểu hotel: Tại sao số khách đến thì ít hơn, nhưng tỉ lệ phòng được lấp đầy lại tăng => Do khách ở lại lâu hơn (Đáp án A)

B. older people: new object

C. E.  Không thể kết luận về xu hướng khi chỉ có 2 data points về 2 thời kì

D. Không kết luận được gì về số lượng khi chúng ta đang có giữ liệu về % lấp đầy và % nhận vào/ 1000 giường

Back

The total market value of real estate in Altonville has steadily declined over the past four years. This decline has meant that the overall figure on which the city’s property tax is based—the assessed value of that real estate—has also declined. Moreover, the percentage of assessed value that was paid as property taxes to the city did not change during this period. 

 

The information above most strongly supports which of the following?


A. Money collected from property taxes provided most of Altonville’s revenue during the past four years.
B. The percentage of Altonville’s overall revenue that was collected from property taxes did not change over the past four years.
C. Altonville officials had proposed raising property tax rates during the past four years but never did so because of strong opposition from property owners.
D. The total amount of revenue that Altonville has collected from property taxes has declined over the past four years.
E. During the past four years, Altonville officials also did not increase tax rates on other sources of revenue such as retail sales or business profits.

Front

*Phải đọc kĩ để ra được 2 ý này

1. Tổng giá trị thị trường của BĐS giảm <=> tổng giá trị định giá BĐS để tính thuế BĐS giảm

2. property taxes = % thuế trên giá trị BĐS không đổi

=> Tổng tiền thu được từ thuế giảm (D)

 

B.  Tỷ lệ thuế BĐS/ tổng thuế của thành phố không giảm

Tổng thuế của thành phố (thuế bđs + thuế kinh doanh + thuế tiêu dùng +...) ko được nói đến

Back

Mud from a lake on an uninhabited wooded island in northern Lake Superior contains toxic chemicals, including toxaphene, a banned pesticide for cotton that previously was manufactured and used, not in nearby regions of Canada or the northern United States, but in the southern United States. No dumping has occurred on the island. The island lake is sufficiently elevated that water from Lake Superior does not reach it.

* Dữ liệu thiếu: Lake Superior is in the northern part of the U.S. and in Canada

The statements above, if true, most strongly support which of the following hypotheses?

(A) The waters of the island lake are more severely polluted than those of Lake Superior.
(B) The toxaphene was carried to the island in the atmosphere by winds.
(C) Banning chemicals such as toxaphene does not aid the natural environment.
(D) Toxaphene has adverse effects on human beings but not on other organisms.
(E) Concentrations of toxaphene in the soil of cotton-growing regions are not sufficient to be measurable.

Front

* Dữ liệu thiếu: Lake Superior is in the northern part of the U.S. and in Canada

* Prompt:

1. Một hòn đảo ở phía Bắc Lake Superior - bị cách biệt khỏi các vùng khác, ko có người ính sống

2. Một cái hồ ở Lake Superior bị ô nhiễm ko rõ từ nguyên nhân nào

3. Chất ô nhiễm giống với chất hoá học được sản xuất và sử dụng ở phía Nam Hoa Kỳ

 

B. -> chất ô nhiễm được chuyển đến nhờ gió

Back
Front

 

D. Đang biện minh cho cái trường này, việc so sánh với trường khác là ko liên quan

Vì dù họ có xấu đến mấy ở mặt A đi chăng nữa thì mặt B họ vẫn có thể tốt để tổng thể chung là họ tốt hơn

 

B. Sự biện minh và assumption của ông hiệu trưởng như vậy là hợp lý.

Tất cả mọi mặt đều tốt, ngoại trừ điểm lớp 5 ko tốt.

=> Ông ấy tin: Trường của ông ấy bị xếp hạng top 50 trường kém nhất CHỦ YẾU LÀ VÌ điểm lớp 5 thấp.

 

Từ đó ông ấy mới kết luận: Theo ông ấy như thế là unfair, vì điểm lớp 5 của trường ông ấy cũng ko phải thấp nhất trong thành phố

 

Back

Boldface

(6 cards)

 

Front

 

Conclusion: Việc scientists ở quá lâu trong 1 lĩnh vực, chứ không phải do tuổi già, khiến khả năng sáng tạo của họ bị giảm

WHY:

- có một lượng lớn scientists vẫn có những công trình sáng tạo dù là đã vượt qua tuổi 40

- những người này tham gia vào các lĩnh vực của họ muộn hơn người khác

Background:

- Common thought: Tuổi già mang đến sự mất khả năng sáng tạo

A. BF2 support, thay vì OBJECT, position defended in the argument.

B. BF2 được sử dụng để against, thay vì support, the position that the argument opposes

C. BF2 against, thay vì support, the explanation that the argument challenges

E. BF1 is challenged by, thay vì defended by,  the argument

D. BF 1 is challenged by the argument + BF2 support the challenge/ the position in the argument

Back

 

Front

 

Conclusion: Bình thường thì KHÔNG BIẾT CHẮC "Việc investor đi CHỈ TRÍCH đội ngũ quản trị là PHẢN TÁC DỤNG"

NHƯNG trong trường hợp này thì chắc chắn "Việc investor đi chỉ trích là KHÔNG HỢP LÝ"

 

WHY: Trường hợp của Burton không phải do sản xuất đang vượt quá demand (BF2)

- Sự tăng lên của inventory là của những đơn hàng đã nhận (đặt trước) từ khách hàng

 

*BF1: not justified: từ ngữ thể hiện 1 cái refutation của argument với thông tin đã được đưa ra trước đó

BF2: It is true that..., but...: đứng ngay sau BF1 để support, giải thích.

Indicates-cũng là từ ngữ thể hiện sự giải thích

Back
Front

 

Conclusion: Bằng chứng "đã tiêu nhiều tiền mà khí thải vẫn tăng" là ko đầy đủ để nói "money spent is wasted"

WHY: - Có 1 số lượng lớn industrial facilities mà xả chất ô nhiễm đã được xây dựng

Background:

- Money spent để mà giảm khí thải độc hại đã bị lãng phí

- Vì tiêu tiền mà khí thải vẫn tăng

A.B. BF1: reasoning ko cố chưng minh là nó sai, mà chỉ bảo nó là "KHÔNG ĐỦ"

E. BF2 ko support BF1, thay vào đó support 1 position against BF1

Back

Plant scientists have used genetic engineering on seeds to produce crop plants that are highly resistant to insect damage. Unfortunately, the seeds themselves are quite expensive, and the plants require more fertilizer and water to grow well than normal ones. Thus, for most farmers the savings on pesticides would not compensate for the higher seed costs and the cost of additional fertilizer. However, since consumer demand for grains, fruits, and vegetables grown without the use of pesticides continues to rise, the use of genetically engineered seeds of this kind is likely to become widespread.

In the argument given, the two portions in boldface play which of the following roles?


(A) The first supplies a context for the argument; the second is the argument's main conclusion.

(B) The first introduces a development that the argument predicts will have a certain outcome; the second is a state of affairs that the argument denies will be part of that outcome.

(C) The first presents a development that the argument predicts will have a certain outcome; the second acknowledges a consideration that weighs against that prediction.

(D) The first provides evidence to support a prediction that the argument seeks to defend; the second is that prediction.

(E) The first and the second each provide evidence to support the argument's main conclusion.

Front

 

Conclusion: Việc sử dụng hạt biến đổi gene này sẽ trở lên phổ biến

WHY: consumer demand for grains, fruits, and vegetables grown without the use of pesticides continues to rise

Counter premise: the seeds themselves are quite expensive, and the plants require more fertilizer and water to grow well than normal ones.

Background: Plant scientists have used genetic engineering on seeds to produce crop plants that are highly resistant to insect damage

B. => sai: BF2 ko phải là một phần của cái outcome của BF1

 

 

Back

Editorial: An arrest made by a Midville police officer is provisional until the officer has taken the suspect to the police station and the watch commander has officially approved the arrest. Such approval is denied if the commander judges that the evidence on which the provisional arrest is based is insufficient. A government efficiency expert has found that almost all provisional arrests meet standards for adequacy of evidence that watch commanders enforce. The expert therefore recommends that the watch commander’s approval should no longer be required since the officers’ time spent obtaining approval is largely wasted. This recommendation should be rejected as dangerous, however, since there is no assurance that the watch commanders’ standards will continue to be observed once approval is no longer required.

In the editorial, the two portions in boldface play which of the following roles?


A. The first is a recommendation made by the editorial; the second acknowledges a potential objection against that recommendation.

B. The first is a proposal against which the editorial is directed; the second is a judgment reached by the editorial concerning that proposal.

C. The first provides evidence in support of a recommendation that the editorial supports; the second is the conclusion reached by the editorial.

D. The first is a position that the editorial challenges; the second is a judgment that was made in support of that challenged position.

E. The first is a recommendation that the editorial questions; the second provides evidence against that recommendation

Front

 

Conclusion: Nên reject recommendation " the watch commander’s approval should no longer be required"

WHY:

- Không có gì đảm bảo "Người ta vẫn giữ standards nếu ko có approval của commander"

B.  BF1 - trái ngược với hướng của bài báo + BF2 - chính là ý kiến của bài báo-> đúng

 

A. BF1 KHÔNG phải recommendation của editorial, mà là của expert.

BF2 KHÔNG acknowledges a POTENTIAL objection, cái objection này đã được đưa ra-KHÔNG CÒN LÀ POTENTIAL

C. BF1 là 1 recommendation, KHÔNG phải là 1 EVIDENCE nữa; editorial cũng ko support cái ý kiến này

D. BF2 - là 1 conclusion, KHÔNG được tạo ra để support cái gì cả; hơn thế nữa, nó cũng ko support cái ý kiến bị challeged này

E. BF2 ko có đưa ra EVIDENCE nào cả, chỉ là 1 cái kết luận

Back
Front

 

Conclusion: AB sẽ tranh cử làm thống đốc bang

WHY:

- Tin đồn trong quá khứ là ông ấy sẽ tranh cử

- Năm nay ông ấy đã thực hiện yêu cầu tiết lộ vấn đề tài chính

- Những người không tin việc ông ấy tranh cử có chỉ ra rằng "tiết lộ vấn đề tài chính là yếu tố cản trở chính đối với ông"

A. lack force= thiếu sức mạnh: nếu thiếu BF1 thì ko rõ tại sao việc ông ấy tiết lộ tài chính liên quan gì đến vấn đề tranh cử

B. BF2 là conclusion của cả bài, không phải là 1 intermediate conclusion

C. BF1 ko phải là 1 practice, nó chỉ là 1 thông tin cung cấp background + premise

BF2: Ko nói đến cái consequence nào của 1 cái practice nào cả

D. BF1 KO đưa ra  evidence bearing against the main conclusion, nó là premise + background -> support

E. BF2 ko phải inter..

Back

Weaken

(34 cards)

 

Front

Conclusion: Bỏ trồng soybean -> chuyển sang trồng cotton sẽ tăng thu nhập trong nhiều năm tới

WHY: Một số vùng cotton đã bị côn trùng phá -> giá cotton tăng

- Cotton thì cũng phát triển rất nhanh

Weaken:

- Sẽ không tăng được thu nhập khi chuyển từ soybean sang cotton đâu?

- Giá cotton vẫn giữ nguyên trong nhiều năm tới hay sao?

 

A. Cost of raising soybeans sẽ tiếp tục tăng -> strengthen

C. Không có sharp increase in the demand for cotton-> ko cần tăng demand, tăng price là đủ rồi

  • Một vài năm trước: không quan tâm

D. ít KH chịu trả giá cao hơn giá cotton hiện tại -> ko cần tăng thêm price, price hiện tại đã đủ cao rồi

E. Loài côn trùng phá cotton - được biết là sẽ ko phá soy bean: Ko liên quan

B. Có những pesticide mới-đã được test và thấy hiệu quả-> các năm sau cotton sẽ được bảo vệ -> cung tăng -> giá giảm -> weaken

Back

In Kravonia, the average salary for jobs requiring a college degree has always been higher than the average salary for jobs that do not require a degree. Over the last few years, the number of Kravonians enrolled in college has been growing steadily. Therefore, the number of Kravonians entering the job market who have at least the qualification of a college degree will eventually be significantly higher than it has been over the last few years.

Which of the following, if true, most seriously weakens the argument?

(A) Kravonians with more than one college degree earn little more, on average, than do Kravonians with only one college degree

(B) The average number of years Kravonian college students remain enrolled before completing a degree has been increasing over the past several years.

(C) Despite the increase in the number of Kravonians attending college, the percentage of the population attending college is lower in Kravonia than in most other countries.

(D) In recent years, employers have been requiring college degrees for workers in jobs that were previously performed successfully by Kravonians who did not have college degrees.

(E) For many years, employers in Kravonia have had difficulty finding enough college graduates to fill the high-paying jobs that were available.

Front

 

Conclusion: Số lượng người có ít nhất 1 bằng đại học tham gia vào thị trường lao động sẽ tăng trong vài năm tới

Why: Vì số lượng người tham gia ghi danh vào đại học tăng (1)

Assumption: Cứ ghi danh tăng là số lượng người học sẽ tăng hay sao?

Prethink weaken: số lượng người có ít nhất 1 bằng đại học tham gia vào thị trường lao động sẽ KHÔNG tăng trong vài năm tới

 

A. Có nhiều bằng cấp thì earn nhiều

C. Tỷ lệ dân số đi học

D. Employers yêu cầu như thế nào

E. Employer thấy khó khăn

-> ko quan tâm

B. Tỷ lệ người tiếp tục ghi danh dù chưa hoàn thành 1 bằng đại học tăng

-> Số lượng người ở (1) chưa chắc đã là những người mới, mà có thể là là người cũ ghi danh thêm

-> Số lượng người mới học đại học chưa chắc đã tăng

-> Weaken

Back

 

Front
Back

The recycling of municipal solid waste is widely seen as an environmentally preferable alternative to the prevailing practices of incineration and of dumping in landfills. Recycling is profitable, as the recycling programs already in operation demonstrate. A state legislator proposes that communities should therefore be required to adopt recycling and to reach the target of recycling 50 percent of all solid waste within 5 years.

Which of the following, if true, most seriously calls into question the advisability of implementing the proposal?

(A) Existing recycling programs have been voluntary, with citizen participation ranging from 30 percent in some communities to 80 percent in others.

(B) Existing recycling programs have been restricted to that 20 percent of solid waste that, when reprocessed, can match processed raw materials in quality and price.

(C) Existing recycling programs have had recurrent difficulties finding purchasers for their materials usually because of quantities too small to permit cost-effective pickup and transportation.

(D) Some of the materials that can be recycled are the very materials that, when incinerated, produce the least pollution.

(E) Many of the materials that cannot be recycled are also difficult to incinerate.

Front

Plan: Yêu cầu các communities đạt target of recycling là 50% solid waste trong vòng 5 năm

Weaken: Ko thể recyling 50% được đâu

A.D.E. Ko liên quan

C. Các chương trình tái chế hiện tại đang gặp khó khăn do số lượng nguyên liệu sau khi tái chế quá nhỏ -> khó tìm người mua do tốn chi phí lấy hàng và vận chuyển

=> Khi mà yêu cầu cả cộng đồng tham gia -> số lượng nguyên liệu sau khi tái chế nhiều -> pickup và transportation sẽ cost-effective hơn

B. Chỉ 20% solid waste sau khi xử lý là đạt yêu cầu chất lượng và giá cả -> Không thể hoàn thành kế hoạch tái chế tận 50%

Back
Front
Back
Front

Conclusion: Đầu tư vào sản xuất giấy sẽ mang lại lợi nhuận

WHY: Vì sản xuất giấy sẽ tạo ra sản phẩm phẩm phụ là lignin + lignin cần cho việc lọc dầu  + giá dầu đang tăng

 

A. Một lượng nhỏ lignin ĐANG (currently) được bán cho công ty hoá chất, nhưng phần lớn bị đốt như là rác

-> Việc ĐANG xảy ra không quan trọng -> tương lai giá dầu tăng, người ta lọc dầu nhiều thì vẫn cần nhiều lignin mà

 

E. Lượng lignin được sản xuất HÀNG NĂM lớn hơn nhiều lần hàm lượng mà SẼ ĐƯỢC (is likely to be) sử dụng trong ngành công nghiệp hoá dầu

-> TƯƠNG LAI dù ngành hoá dầu sử dụng lignin thì cũng chỉ là một lượng nhỏ -> không có profit cho investors đầu tư vào sản xuất giấy

 

 

Back
Front

NOTE: KO chọn big words - distraction

Conclusion: Ít women đắc cử, không phải do women có khó khăn trong việc đắc cử mà do ÍT women MUỐN run for state and national offices (1)

Why:

- Vì khả năng thắng cử của women và men là như nhau

- Chỉ 15% ỨNG CỬ VIÊN cho các offices này là women

 

A. Tỷ lệ người đương nhiệm TÁI ĐẮC CỬ ->mình quan tâm đến tỷ lệ ĐẮC CỬ NÓI CHUNG chứ không quan tâm đến tỷ lệ TÁI ĐẮC CỬ/ ĐẮC CỬ MỚI

 

E. Women MUỐN run for state and national offices NHƯNG không có đủ fundding

=> số lượng women muốn run for offices VẪN RẤT NHIỀU >< (1)

=> Weaken conclusion

Back
Front

Conclusion: Thiếu cạnh tranh nên bank ở Cali có lãi suất cho vay cao hơn ở các vùng khác ở Mỹ

Prethink: Có 1 nguyên nhân khác dẫn đến việc lãi suất cho vay cao hơn

 

A. Weaken: Bởi vì lương ở Cali cao hơn -> Banks ở Cali phải charge phí dịch vụ cho người gửi tiết kiệm cao hơn.

=> Lương cao hơn cũng có thể cùng là nguyên nhân cho việc banks ở Cali phải set lãi suất cho vay cao

D. Strengthen: Ít người ở Cali không trả được nợ vay so với các vùng khác => Loại trừ được lý do khác - "do nhiều người không trả được nợ" dẫn đến lãi suất cho vay cao => strengthen

Back

Editor: Articles in Gardening Magazine often spur sales of the plants they describe, particularly among people new to gardening. Accordingly, we will no longer publish articles or accept advertisements praising the beauty of rare wildflowers. Most such plants sold to gardeners have been difficult to propagate under cultivation, so plant sellers often collect them in the wild. Our new policy is part of our efforts to halt this yearly plundering of our native plant populations.

Which of the following, if true, casts the most doubt on the wisdom of the magazine's new policy as a way of pursuing the intended effect?

(A) When people new to gardening buy plants, they often fail to take adequate care of the plants that they buy and become discouraged from buying those varieties again.

(B) Plant sellers who sell rare wildflowers have no reasonably inexpensive alternate way to offer their wares directly to new gardens.

(C) The demand for rare wildflowers rarely exceeds the number of such plants that can be collected in the wild by plant sellers.

(D) The propagation of rare wildflowers often depends on the plant's interaction with other organisms in their environment such as plants that create suitable soil conditions or insects and birds that disperse seeds.

(E) Revenues from sales of plants collected in the wild are supporting the discovery of new low-cost techniques enabling rare wildflowers to be readily propagated in nurseries.

Front

Conclusion: Việc KHÔNG publish articles hoặc không chấp nhận quảng cáo các loài hoa dại quý hiếm SẼ LÀM GIẢM 1/2 số lượng "cướp bóc" các loài thực vật bản địa

WHY: Vì các loài hoa này rất khó nhân giống (propagate), nên người bán hoa thường thu thập từ bên ngoài tự nhiên

Weaken:  Việc KHÔNG publish articles hoặc không chấp nhận quảng cáo SẼ KHÔNG LÀM GIẢM số lượng "cướp bóc" các loài thực vật bản địa ĐÂU, VÌ SAO?

A. Việc chăm những cây trồng đã được mua (ko chăm sóc đủ ->bị chết)

A. họ bị discouraged from buying -> strengthen kế hoạch

B. CHI PHÍ của việc có hàng hoá giao cho khách

C. Việc plant sellers cướp bóc ĐỦ/ KHÔNG ĐỦ số hoa quý để phục vụ demand

D. Lý do khó để nhân giống hoa dại quý hiếm

-> ko liên quan đến việc có giảm được số lượng cây không

Và: Do việc nhân giống rất khó/ chi phí cao & cầu<cung -> họ sẽ đi lấy hoa từ bên ngoài nếu có người mua -> bỏ bớt ads thì sẽ bớt được cầu -> hoa bị hái

 

 

E. Rev từ việc bán hoa giúp nghiên cứu các phương pháp nhân giống hoa

-> nếu không có rev từ đây thì sẽ không có hoa nhân giống và plant sellers lại càng phải đi lấy hoa từ ngoài tự nhiên 

Back
Front

Pattern: Cái hiện tại đã quá đủ thì Cái mới có công năng tốt hơn cũng không improve được chất lượng

 

Conclusion: Newtape sẽ improve healing

WHY: Newtape giữ vết thương lâu gấp đôi so với loại tape hiện tại đang sử dụng

Các loại tape này đều được dùng để giữ các vết thương phẫu thuật trong 10 ngày - thời gian lâu nhất mà vết thương cần tape

Prethink: Newtape không improve được gì cả.

 

A. About ten days: Khoảng 10 ngày-> Vậy cụ thể là có bao nhiêu % phẫu thuật cần hơn 10 ngày

Không giúp để support hay weaken argument

E. Đề bài chỉ bàn về THỜI GIAN KẾT DÍNH, ko nói về việc có chất hoá học ở da hay không

-> E đang cố gắng break the premise của đề bài -> Sai. Premise luôn luôn đúng.

 

C. Tape hiện tại đã quá đủ để giữ vết thương trong 10 ngày-> Mà mục đích duy nhất của loại tape này là để giữ các loại vết thương lành lại trong 10 ngày => không cần loại tape có thời gian giữ lâu hơn

Back
Front

Conclusion: Giảm giá trị đồng tiền -> tăng xuất khẩu

WHY:

- Trước đây có 2 lần: Giảm giá trị đồng tiền -> tăng xuất khẩu

- Giảm giá trị đồng tiền -> khiến giá sản phẩm rẻ hơn trên thị trường thế giới -> Tăng xuất khẩu

Weaken: Giảm giá trị đồng tiền cũng KO tăng xuất khẩu đâu, tại sao?

A. politicians- trước đây recommend- giờ cũng recommend -> strengthen. Có lý do để nghe theo mấy ông này

C. The economy có trở lên healthier hơn hay ko -> ko liên quan

D. Các nước đang compete có stable currencies -> ko liên quan

E. Có cách khác để giảm giá hàng hoá -> ko liên quan

B. Sản xuất đã đạt đỉnh => không thể có thêm hàng hoá để xuất khẩu

 

Back
Front

Conclusion: 50.000 hotel room night bookings giảm không phải do website Miami@50 mà là do nền kinh tế

Why: Vì Miami@50 chỉ rented out (cho thuê) 8000 apartments

 

Weaken: chính vì 8000 apartments này cho thuê mà nó giảm 50.000 hotel room night bookings đấy

Notes: Chú ý vào reasoning - ở đây quan tâm đến 8000 apartments của Miami@50 chứ không quan tâm đến hotel room night booking của Miami@50

 

B. Miami@50 chỉ nói cho thuê 8000 apartments, nhưng không nói là book bao nhiêu apartments => số hotel room night bookings của Miami@50 dù gấp 8 lần số apartment bookings thì cũng không rõ là bao nhiêu

=> ko weaken.

C. Đa số apartments được book nhiều lần => 8000* 7 lần là đã lớn hơn 50.000 rồi => weaken.

Back
Front

Conclusion: Giữ teenager ở nhà vào buổi tối không làm giảm số lượng SERIOUS crimes

Why: Vì đa số crimes committed by teenagers thì xảy ra vào lúc 3pm-6pm

 

PRETHINK_Weaken: Giữ teenager ở nhà vào buổi tối SẼ LÀM giảm số lượng SERIOUS crimes, tại sao?

 

A. Ko liên quan. Ở đây đã chốt điều kiện "teenagers bị giữ ở nhà rồi"

C. Không đủ weaken. Buổi tối-lúc ở nhà- thì commit ít crimes, nhưng lúc 3-6pm thì vẫn commit nhiều hơn trước kia thì sao?

D. Irrelevant/ Strengthen: Việc tuần tra tăng hay không ảnh hưởng gì đến việc phạm tội? Ở đây tuần tra không tăng thì làm sao mà giảm được phạm tội?

E. Chỉ học đến 6 giờ vào weekday afternoons, vậy còn weaken thì sao?

B. Crimes commited vào buổi chiều không quan trọng -> ko phải serious crimes -> Buổi tối giảm được SERIOUS là ok rồi

Back

Escalating worldwide demand for corn is expected to cause the market price of corn to rise sharply and remain high. Although corn is extensively used as feed for livestock, livestock feed accounts for only a small fraction of the retail price of meat. Therefore, the increase in corn prices is very unlikely to produce any comparable long-term increase in the retail price of meat.

Which of the following, if true, most seriously weakens the argument?


(A) The cost of shipping and handling meat has also risen sharply.

(B) Livestock producers who currently use corn to feed their livestock have the option of switching to other kinds of feed.

(C) The rising cost of feed is leading some livestock producers to leave the business, thereby reducing the future supply of meat.

(D) Worldwide demand for grains other than corn has also increased.

(E) The price of corn affects the retail price of a variety of other food products more than it affects the price of meat.
 

Front

Conclusion: Giá ngô tăng KHÓ CÓ KHẢ NĂNG tạo ra sự tăng giá thịt lợn TRONG DÀI HẠN

Why: Giá ngô chỉ chiếm 1 phần nhỏ của giá bán lẻ thịt lợn

Weaken: Phải liên quan đến các yếu tố trong conclusion "Giá ngô và giá thịt lợn dài hạn"

 

A. Cost of shipping and handling meat: Ko liên quan đến conclusion  "Giá ngô và giá thịt lợn dài hạn"

C. Đáp án duy nhất nói đến "giá thịt lợn TRONG DÀI HẠN"

 

B. Strengthen.

D. E. Không liên quan

Back
Front

 

Conclusion: Giảm lương tối thiểu cho teenager -> giảm thất nghiệp cho teenager

Assumption: Có mối quan hệ giữa lương tối thiểu và tỉ lệ thất nghiệp ở teenager

Prethink: Không có mối quan hệ nào giữa lương tối thiểu và tỉ lệ thất nghiệp ở teenager

 

B. Tỉ lệ thất nghiệp của teenager tăng cả khi lương tối thiểu giữ nguyên -> không có mối quan hệ cause-effect nào ở đây

-> Weaken

Back

Some species of dolphins find their prey by echolocation; they emit clicking sounds and listen for echoes returning from distant objects in the water. Marine biologists have speculated that those same clicking sounds might have a second function: particularly loud clicks might be used by the dolphins to stun their prey at close range through sensory overload.

Which of the following, if discovered to be true, would cast the most serious doubt on the correctness of the speculation described above?


(A) Dolphins that use echolocation to locate distant prey also emit frequent clicks at intermediate distances as they close in on their prey.

(B) The usefulness of echolocation as a means of locating prey depends on the clicking sounds being of a type that the prey is incapable of perceiving, regardless of volume.

(C) If dolphins stun their prey, the effect is bound to be so temporary that stunning from far away even if possible, would be ineffective.

(D) Echolocation appears to give dolphins information about the richness of a source of food as well as about its direction.

(E) The more distant a dolphin's prey, the louder the echolocation clicks must be if they are to reveal the prey's presence to the hunting dolphin.

Front

Phỏng đoán: Cái âm thanh dolphins phát ra để tìm con mồi cũng có một tác dụng thứ 2 là - stun con mồi ở gần thông qua việc sensory overload

Weaken: Cái âm thành dolphins phát ra để tìm con mồi KHÔNG CÓ TÁC DỤNG  stun con mồi ở gần thông qua việc sensory overload ĐÂU, TẠI SAO?

Prethink: phải liên quan đến "việc stun con mồi" và "việc dolphin phát ra âm thanh", nếu không liên quan thì kiểu gì cũng sai

 

A. Dolphin phát ra âm thanh giống nhau ở cả khoảng cách trung bình và khoảng cách gần -> ko liên quan đến việc stun con mồi

B. Con mồi không thể hấp thụ âm thanh, dù lớn cỡ nào -> con mồi không thể cảm nhận được nó chứ chưa cần nói đến bị overload

Back

 

Front

Kế hoạch: Toà án tối cao cho phép các công ty từ chối ứng viên, nếu công việc gây ra 90% khả năng là ứng viên đó sẽ bị đau tim

-> từ đó bảo việc cả ứng viên và nhà tuyển dụng

Weaken: Đạo luật này sẽ không hiệu quả trong việc điều tiết hành vi tuyển dụng.

Prethink: Đáp án phải liên quan đến % ứng viên bị đau tim và lợi ích đối vơi ứng viên + nhà tuyển dụng

A. 1 cái fact, không liên quan

*Không được assume rằng: vì người ta không muốn tuyển nên người ta sẽ nói là ứng viên có % bị đau tim cao

B. Không có phương pháp đo lường risk -> Đánh TRỰC TIẾP vào kế hoạch -> Kế hoạch không thành công

E. - Might: ko rõ là có xảy ra không

- Lượng người apply giảm -> càng tốt, lọc được bớt những đứa yếu tim

Back
Front

Banning (nghiêm cấm) # restrictions (hạn chế)

 

D. Khi restrictions, restaurant có thể sắp xếp các separate dining areas mà smoking is permitted

Nhưng khi banning thì cấm toàn bộ việc smoking

=> Weaken việc tăng sales của restaurants

 

E. Sales taxes nói chung của các towns (retail taxes, property taxes,..)

Ko liên quan cụ thể đến restaurant taxes

Back
Front

Conclusion: 20% các stores bị yếu kém trong vòng 5 năm tới không thể vực dậy được

Mặc dù: Trước đây cũng từng có 20% cửa hàng bị yếu kém

Nhưng: Giờ các cửa hàng bị yếu kém là vì compete trực tiếp với SaveMart

 

Tại sao? Prethink: SaveMart ảnh hưởng như thế nào khiến các cửa hàng không thể vực dậy được?

A. Các cửa hàng bị đóng cửa vì Savemart là các cửa hàng mỏ neo. Mà khả năng kéo khách hàng của các cửa hàng khác phụ thuộc vào các cửa hàng mỏ neo này-> không thể vực dậy được 20% các stores bị yếu kém này

 

D. Kế hoạch của SaveMart dù có là lợi nhuận thấp đi chăng nữa cũng ko biết ảnh hưởng đến các stores này như thế nào

KO ĐƯỢC ASSUME: lợi nhuận thấp thì để giá thấp để cạnh tranh mạnh với các cửa hàng khác -> họ ko vực dậy được

 

E. Opposite. Shrink (co ngắn lại) thay vì collapse -> vẫn còn khả năng rebound.

Back


 

Front

Conclusion: Nếu bỏ thuế quan xuất khẩu -> tăng thất nghiệp ở thành thị

Why: nông dân sẽ bán hạt điều chưa xử lý ra nước ngoài vì như thế sẽ có nhiều lợi nhuận hơn -> nhà máy ở thành không có nguyên liệu -> giảm lao động

Weaken: Tăng thuế quan lên cũng ko giảm được tỉ lệ thất nghiệp ở thành thị đâu

E. Tăng thuế quan xuất khẩu -> nông dân không có lợi nhuận -> lên thành phố để làm việc và thất nghiệp ở thành thị sẽ tăng

Back
Front

 (postage rates)= shipping costs

issues: số lượng tờ báo được phát hành trong 1 năm

Conclusion: Tăng profit bằng cách
- Giữ nguyên giá subscription
- Giảm phí bưu cục bằng cách giảm 1/2 số lượng issues

- Neither subscribers nor advertisers will be lost

 

Weaken plan: Profit sẽ bị giảm

A. Số lượng issue giảm 1/2 mà giá bưu cục chỉ tăng 1/3 => vẫn giảm được cost

B. Subcribers quan tâm nhiều về quality mà quality vẫn được giữ nguyên -> ko có vấn đề gì

C. Nhiều người sẽ vẫn tiếp tục subcribe -> Good

E. Cost giữ nguyên -> Good

D. Không bị mất đi Advertisers tuy nhiên doanh thu từ đây sẽ giảm đi 1/2 do:  Advertisers vẫn giữ nguyên số tiền quảng cáo trung bình/ 1 issue mà số lượng isue giảm 1/2

Back
Front

Conclusion: Năm ngoái, team dược phẩm tăng trưởng tốt

Why: Năm ngoái team team hoá học, bình thường chiếm 60% profits, tăng trưởng kém. (1)

- Năm ngoái team dược phẩm đã chiếm 45% profits rồi (tăng từ 20% của năm trước đó) (2)

Assumption: % trong tổng profits tăng đâu có nghĩa là profit của team dược phẩm tăng đâu

 

Weaken: Những bằng chứng trên chả có ý nghĩa gì để nói team dược phẩm tăng trưởng tốt cả, tại sao?

Note: Đọc kĩ conclusion. Không lý gì conclusion lại bao gồm cả bài (nói về cả hoá học và dược phẩm được).

(1) chỉ cố củng cố (2) mà thôi. (Dù team kia giảm % nhưng team tôi vẫn tăng % nè)

 

A. Increase đến từ đâu new product

B. Ở các công ty khác, 50% profits đến từ dược phẩm

=> không liên quan đến việc kết luận là team nào tăng trưởng tốt/ không tốt ở đây

D. Mình không quan tâm 20% của năm xưa có phải là một improvement SO VỚI NĂM TRƯỚC ĐÓ  hay không, mình chỉ quan tâm năm ngoái so với năm xưa

E. Kế luận không liên quan đến việc "compare 2 divisions với nhau"

C. Weaken: % trong tổng profit của team dược phẩm tăng kể cả khi perfomance của nó không tăng

=> Khả năng là tổng profit của công ty bị giảm =>% trong tổng Profit của team dược phẩm đã tăng lên

=> Pharmaceutical division is NOT growing stronger

 

Back

Nalmed Province's plan is to reduce highway congestion by expanding the commuter rail system, so giving more people the option of travelling by train. When a recent opinion poll presented this plan to province residents, they overwhelmingly favored it, even though they knew that enacting the plan would mean substantial tax increases. Consequently, the plan, if enacted, is very likely to succeed, because if the people are prepared to pay, they expect to reap the benefit.

The inference made from the poll results is most vulnerable to criticism on the grounds that
A) it overlooks the possibility that once the highways are consistently uncongested, some commuters might have no motivation to switch from using their car to using the rail system.
B) the favorable responses collected are entirely consistent with every one of those respondents expecting that it would be others who, by using the rail system, would ease highway congestion.
C) those respondents who opposed the plan might nevertheless become users of the rail system as a result of its expansion.
D) those respondents who opposed the plan might oppose it for reasons other than the tax increase required to carry it out.
E) residents responding to the poll are likely to overestimate the tax increase they themselves will experience if the proposed expansion occurs.
 

Front

Conclusion: Kế hoạch giảm high way congestion bằng mở rộng rail system của tỉnh sẽ  thành công
Backgrounds: Tỉnh thu tiền thuế để làm rail system và thấy đa số người dân đồng ý trả tiền
Assumption: Province nghĩ là người đồng ý trả tiền thì người ta sẽ dùng rail system 
Prethink: Không giảm high way congestion bằng railroad được, dù có nhiều người đồng ý trả tiền

- Người ta đồng ý trả tiền nhưng người ta không đi train thì sao

 

B. Những responder nghĩ rằng những người khác mới using rail system chứ không phải bản thân họ.

Back

Damaged nerves in the spinal cord do not regenerate themselves naturally, nor even under the spur of nerve-growth stimulants. The reason, recently discovered, is the presence of nerve-growth inhibitors in the spinal cord. Antibodies that deactivate those inhibitors have now been developed. Clearly, then, nerve repair will be a standard medical procedure in the foreseeable future.

Which of the following, if true, casts the most serious doubt on the accuracy of the prediction above?


(A) Prevention of the regeneration of damaged nerves is merely a by-product of the main function in the human body of the substances inhibiting nerve growth.

(B) Certain nerve-growth stimulants have similar chemical structures to those of the antibodies against nerve-growth inhibitors.

(C) Nerves in the brain are similar to nerves in the spinal cord in their inability to regenerate themselves naturally.

(D) Researchers have been able to stimulate the growth of nerves not located in the spinal cord by using only nerve-growth stimulants.

(E) Deactivating the substances inhibiting nerve growth for an extended period would require a steady supply of antibodies.

Front

 

Conclusion: Việc sửa chữa dây thần kinh sẽ là một phương pháp y học tiêu chuẩn trong tương lai gần

Why: Đã phát hiện ra antibodies (kháng thể) - cái mà có thể deactivate cái inhibitor (cái ngăn chặn việc dân thần kinh - bị hư hại -  tự phục hồi)

 

Prethink weaken:  Việc sửa chữa dây thần kinh sẽ KHÔNG PHẢI là một phương pháp y học tiêu chuẩn trong tương lai gần

B. SIMILAR: cấu trúc hoá học của cái simulants chỉ là TƯƠNG TỰ THÔI với antibodies thôi

-> không thể giúp lý luận là cái antibodies không hiệu quả

C. Dây thần kinh ở trong não tương tự dây thần kinh ở trong spinal cord (cột sống)

D. Có thể stimulate the growth of nerves not located in the spinal cord

E.  require a steady supply of antibodies

-> không liên quan

A. Cái inhibitors nó có main function khác -> deactivate nó sẽ stop cái main function của 1 bộ phận trong cơ thể con người

-> Không biết việc stop này có ảnh hưởng nghiêm tọng đến mức nào 

-> Không chắc là nó sẽ trở thành 1 phương tiêu chuẩn trong tương lai 

-> Weaken

Back

After observing the Earth’s weather patterns and the 11-year sunspot cycle of the Sun for 36 years, scientists have found that high levels of sunspot activity precede shifts in wind patterns that affect the Earth’s weather. One can conclude that meteorologists will be able to improve their weather forecasts based on this information.

 

Which of the following, if true, most seriously weakens the argument above?

 

(A) Weather forecasts are more detailed today than they were 36 years ago.

(B) Scientists can establish that sunspot activity directly affects the Earth’s weather.

(C) Evidence other than sunspot activity has previously enabled meteorologists to forecast the weather conditions that are predictable on the basis of sunspot activity.

(D) Scientists have not determined why the sunspot activity on the Sun follows an 11-year cycle.

(E) It has been established that predictable wind patterns yield predictable weather patterns.

Front

Keyword: IMPROVE

Conclusion: meteorologists IMPROVE their weather forecasts (không chỉ là forecast được mà forecast phải CHÍNH XÁC HƠN).

WHY: Vì scientists phát hiện ra là luôn có high levels of sunspot activity xảy ra trước shifts in wind patterns that affect the Earth’s weather

Assumption: Cứ tìm ra được một cái gì mới là người ta có thể IMPROVE được cái cũ hay sao?

Prethink: Meteorologists sẽ KO IMPROVE được their weather forecasts đâu

 

C. Có những dẫn chứng khác ngoài sunspot activity ĐÃ GIÚP meteorologists DỰ ĐOÁN được cái điều kiện thời tiết - mà có thể dự đoán bởi sunspot activity

=> sunspot activity cùng lắm là tương đương so với những công cụ trước đây => KO có IMPROVE => Weaken

Back
Front

 

E. Tất cả các loại thuốc: Mua gói lớn thì rẻ hơn mua liều lượng nhỏ

=> ko liên quan, không giúp được gì

 

D. Cùng thành phần -> có thể có tác dụng tương đương + mà giá rẻ => Đây mới là best buy chứ không phải SinEase

Back
Front

Conclusion: nếu major industries tăng capital reserves, the employment rate will NOT decline in the future.

<=> Nếu có X thì Y chắc chắn không xảy ra

 

WHY: the employment rate giảm vì có predictions of slow economic growth

- Mà nếu có capital reserves thì predictions sẽ không ảnh hưởng được the employment rate

 

Prethink_weaken: Dù có capital reserves, the employment rate vẫn có thể giảm

<=> Dù có X, Y vẫn có thể xảy ra

 

C. Một nguyên nhân khác, ngoài X, vẫn có thể dẫn tới Y=> weaken

Back
Front

The conclusion is:- " it is always a change for the worse to replace packaging made from paper or cardboard with packaging made from plastics that are not biodegradable in landfills.

 

The correct option should show that " it's not always a change for the worse to replace packaging made from paper or cardboard.

Option A shows that paper and cardboard are also bad and so.

 

B. Ko quan tâm đến plastics mà biodegradable

Bài chỉ bảo thay thế paper or cardboard with packaging made from plastics that are not biodegradable in landfills.( plastics mà chắc chắn là KO biodegrable)

Back
Front

 

Conclusion: Thay ghế cao hơn dẫn đến profit sẽ tăng

Why:

- Mọi người đến đây để thấy celebs (1)

- Mọi người muốn có bàn và ghế cao để có tầm nhìn celebs tốt hơn (2)

- Diners ở bàn ghế cao sẽ không ngồi lâu như ở bàn ghế thường (3)

 

Assumptions:

- Bàn ghế cao có khiến người ta chi ít hơn? (4)

- Mọi người có thực sự muốn nhìn celebs và ăn uống?

Weaken:  Thay ghế cao hơn SẼ KHÔNG dẫn đến profit sẽ tăng, tại sao?

 

A. Celeb chọn ngồi ở đâu

B. Giá bữa ăn của celeb

E. Chỉ có bàn cao khác mới có tầm nhìn

-> ko liên quan

D. Người ăn nhanh sẽ tiêu ít tiền hơn-> hơi weaken, nhưng chưa hẳn break argument. Tuy tiêu ít tiền hơn nhưng số lượng khách ra vào tăng lên (do người ta không ngồi lâu) -> Vẫn có thể tăng profit

C. Break argument (Most weaken): Những người ở bàn ghế cao này sẽ ngồi lâu (an exception to the generalization about lingering) -> ko dẫn đến việc tăng số lượng turn over

Back
Front

 

*publicity: sự công bố

Conclusion: số lượng tai nạn máy bay không tăng lên đâu, mà do số lượng các báo đưa tin tăng lên đấy

Why: Vì cứ có tai nạn máy bay là báo chí sẽ tập trung vào ngành máy bay -> tăng số lượng các nguồn mới đưa tin về tai nạn máy bay

 

ACD. công bố thông tin giới hạn trong nước/ tổ chức mới không có guideline/ New source chỉ cover nếu thấy nó có lợi

=> Đều ko ảnh hưởng gì đến kết luận"số tai nạn máy bay có tăng lên không"

E. Strengthen: Số lượng miles không đổi -> khó có thể tăng số lượng tai nạn lên được

B. Weaken: Có nhiều accidents ở các peak travel months đây còn gì => Đây chính là nguyên nhân dẫn đến tăng số lượng reported accidents.

Tại sao officials lại dám nói là accidents không tăng. Bố láo

 

Back
Front

 

Conclusion: Phương pháp của X loại bỏ đối thử bằng cách giảm giá như thế này không thể có lãi trong dài hạn

Why: Lúc nào X tăng giá để bù đắp cái losses trước đó thì các đối thủ sẽ giảm giá thấp hơn giá của X (undercut the airline's fares)

Prethink: X vẫn có thể có lãi trong dài hạn

 

A.C. some/ sometimes

A.C.E legal hay not legal/ giảm giá dưới mức duy trì kinh tế/ số lượng passengers tăng/ các hãng đối thủ shift resources : ko liên quan

D. Ko được assumption: tất cả các hãng đối thủ của X đều bị đẩy ra khỏi routes này.

Vẫn có thể có một số hãng còn tồn tại và giảm giá thấp hơn giá X mà

- Tập trung vào conclusion: Có thể có lãi trong dài hạn hay không???

B. Competitors mà giảm giá thấp hơn giá của X -> X lại giảm giá để đẩy các hãng này ra sau đó tăng giá trở lại -> vẫn có thể có lãi

Back

Situation: For five years the Souper restaurant chain has maintained rapid sales growth in Danport, primarily by opening new restaurants that draw patrons away from other restaurants in their vicinity.

Goal: Souper wishes to achieve continued rapid sales growth in Danport over the next two years.

Proposal for consideration: Continue to open new restaurants in Danport during the next two years at the same rate as in the last two years.

In light of the situation, which of the following, if true, most strongly argues that adopting the proposal would be an ineffective way of achieving the goal?

(A) At times at which customers find Souper restaurants too crowded, they often go to other restaurants nearby.

(B) The Souper chain has generally opened new restaurants in locations that are in the vicinity of a large number of other restaurants.

(C) Souper restaurants generally offer a much smaller variety of foods than many of the other restaurants in their vicinity.

(D) Virtually all potential sites for new Souper restaurants in Danport are located in the vicinity of existing Souper restaurants.

(E) Souper restaurants have always offered meals that are somewhat less expensive than meals at restaurants that compete with Souper for patrons.

Front

 

B. Souper NÓI CHUNG (generally) là đã mở cửa hàng ở gần 1 SỐ LƯỢNG LỚN các cửa hàng khác

-> MỘT SỐ LƯỢNG LỚN + NÓI CHUNG -> Không xác định là bao nhiêu % -> Gần như chỉ lặp lại câu 1 trong argument

-> vẫn có thể mở nhiều cửa hàng Souper mới

 

D. Gần như toàn bộ các cửa hàng tiềm năng đều nằm gần các cửa hàng Souper hiện tại

-> Nếu mở cửa hàng ở các khu vực tiềm năng này thì các cửa hàng mới sẽ "eat  into revenue" của các cửa hàng hiện tại

Back

Researchers have concluded from a survey of people aged 65 that emotional well-being in adulthood is closely related to intimacy with siblings earlier in life. Those surveyed who had never had any siblings or who said that at college age they were emotionally distant from their siblings were emotionally less well adjusted at 65 than were those who had been close to at least one brother or sister.

Which of the following, if true, most seriously weakens the researchers’ argument?


(A) As they get older, many people think more about their mortality and thus must confront feelings of loneliness and isolation.

(B) People suffering from the emotional distress of maladjustment usually remember being less intimate with other people than they actually were.

(C) Memory of one’s past plays a greater role in the emotional well-being of older people than it does in that of younger people.

(D) Few people can correctly identify the true sources of their emotional well-being or of their emotional difficulties.

(E) Siblings are more likely to have major arguments and deep differences of opinion at college age than at any other time of their lives

Front

 

Conclusion: Gần gũi anh chị em hồi trẻ -> Tuổi già sẽ có cảm xúc tốt

WHY: Không gần gũi anh chị em hồi trẻ -> Điều chỉnh cảm xúc kém hơn

Prethink: Đây chỉ là coincidence, có nguyên nhân khác ảnh hưởng đến điều chỉnh cảm xúc

 

B. Weaken: Những người trải qua sự đau buồn do mất cân đối cảm xúc (maladjustment) thường nhớ là họ ít gần gũi với mọi người hơn thực tế

-> những người điều chỉnh cảm xúc kém có thể họ đã nghĩ là họ ít gần gũi với anh chị em, nhưng thực tế là không phải vậy. Họ có gần gũi nhưng họ không nhớ

D. Xác định nguồn của their emotional well-being or of their emotional difficulties -> ko liên quan.

Cần quan tâm đến mối quan hệ giữa their emotional well-being và sự gần gũi với anh chị em

Back
Front

a very common flaw of reasoning:

Fact: X leads to A

Fact: Y leads to A

Conclusion: X = Y.

This is wrong. Same ending does not mean same beginning.

 

Conclusion: Điều kiện nhiệt độ và áp suất cần có để tổng hợp fullerenes sẽ giúp chúng ta biết được trạng thái vỏ Trái Đất tại thời điểm "fullerenes trong tự nhiên" được hình thành

WHY: fullerenes yêu cầu nhiệt độ và áp suất đặc biệt để hình thành

Assumption: fullerenes tổng hợp và fullerenes trong tự nhiên là giống hệt nhau?

Prethink: Điều kiện khi tổng hợp fullerenes KHÔNg GIÚP chúng ta biết trạng thái vỏ Trái Đất tại thời điểm "fullerenes trong tự nhiên" được hình thành đâu

 

D. Weaken. fullerenes trong tự nhiên có cấu trúc chưa được biết -> làm sao biết fullerenes trong tự nhiên có giống fullerenes tổng hợp không. => ko biết được điều kiện hình thành 2 thứ có giống nhau không.

Back

Strengthen

(11 cards)

Electronic computer chips made of tiny silicon wafers now regularly contain millions of electronic switches. Unfortunately, electronic switches that are this small cannot withstand intense radiation. Micro-Mechanics plans to produce a chip that, because it uses only microscopic mechanical switches, will be invulnerable to radiation damage. The switches will, however, be slower than electronic switches and the chip will contain only 12,000 switches.

For there to be a market for Micro-Mechanics’ chip as a result of the apparent advantage described above, each of the following would have to be true EXCEPT:

(A) There will be applications in which the speed attainable by an electronic switch is not essential.

(B) Switches used on electronic chips that contain only 12,000 switches are more vulnerable to radiation damage than the switches on Micro-Mechanics’ chip will be.

(C) There will be applications for computer chips in environments where the chips may have to survive intense radiation.

(D) Some devices in which computer chips will be used will have other components that will be able to function during or after exposure to radiation.

(E) Manufacturers are able to protect electronic computer chips against exposure to intense radiation, where this protection is necessary.

Front
Back
Front

 

Conclusion: Sự ra đời của giống lúa mì mới đã DẪN ĐẾN việc tạo ra bánh mì bột nở

Why: Vì sự ra đời của giống lúa mì này trùng thời điểm với sự phát hiện ra bánh mì bột nở

- Loại lúa mì này được prefer hơn vì nó có thể tách nhân khỏi hạt mà không phải nướng trước

Strengthen: Việc nhân có thể tách ra khỏi hạt mà không phải nướng có lợi ích gì?

 

C. Không nướng -> giúp giữ lại protein cần có cho yeast

Back

For several years Buddy's Bolt Barn has specialized in supplying fasteners - nuts , bolts, screws - to workshops and manufacturing firms in the region . Buddy's in fact now supplies virtually all of the fasteners that this segment of the region's industry uses . Recently , however , many of the region's workshops and manufacturing firms have closed down .Therefore , unless Buddy's can expand into a new market segment , it too is likely to go out of business.

Which of the following , if true , most strengthens the argument ?

a. Most of the fasteners that Buddy's carries in inventory are not specialized and can be used in a wide variety of industries .

b. Historically , period with a lot of closures of workshops and manufacturing firms have generally been followed by periods with many startups of such firms.

c. Even with the recent closures , there are more workshops and manufacturing firms operating in the region now than there were when Buddy's Bolt Barn first opened.

d. The workshops and manufacturing firms in the region that have remained in business have all laid off workers because of shortage of orders.

e. Some of Buddy's managers have prior retail experience in other market segments.

Front

- Conclusion: Công ty X sẽ bị phá sản nếu không mở rộng market segment
- WHY: Một số công ty ở segment hiện tại - workshops and manufacturing firms - đã closed down

- Assumption: Buddy's không thể bán thêm được hàng cho các công ty chưa closed down để bù đắp được hay sao?
Pre think: Buddy's sẽ bị phá sản thôi

B. Historically, period with a lot of closures of workshops and manufacturing firms have generally been followed by periods with many startups of such firms: 
-> Thời kì đóng cửa các workshops và các manufacturing firms này sẽ followed bởi thời kì hình thành các startups tương tự -> có startups mới mua hàng thì công ty X đâu có bị phá sản

D. The workshops and manufacturing firms in the region that have remained in business have all laid of workers because of shortage of orders.
-> Các workshops và manufacturing firms còn sống thì phải cho công nhân nghỉ việc vì thiếu đơn hàng
-> Các workshops và manufacturing firms này cũng sẽ dần đóng cửa -> công ty X kiểu gì cũng bị phá sản

 

1. Nhai từng chữ, đọc cẩn thận tránh hiểu sai câu B ( công ty lớn closed down thì các startups cũng closed :( )
2. Double check lại đáp án đúng, contender

Back
Front

Conclusion: Kế hoạch giết những con nema này sẽ thành công

WHY: Spread chemicals on the fields sẽ khiến những con nema này tỉnh dậy và chết đói

(Assumption: Nó chỉ ăn khoai tây)

 

B. Phần duy nhất của cây khoai tây mà con nema này ăn là dễ cây

=> Mình đâu biết dược nó có ăn cái gì khác không?

=> Ko có support được gì

D. 9/10 con nema sẽ tỉnh dậy và chết đói mà chỉ cần 1 lượng nhỏ chemical-> Thành công rất là cao

Back
Front

A. Sau 1930 thì bọn này mới trao đổi hàng hoá thì cũng ko chứng minh được cái camps này hình thành trước 1930 hay sau 1930

Back

Personnel officer: The exorbitant cost of our health-insurance benefits reflects the high dollar amount of medical expenses incurred by our employees. Employees who are out of shape, as a group, have higher doctor bills and longer hospital stays than do their colleagues who are fit. Therefore, since we must reduce our health-insurance costs, we should offer a rigorous fitness program of jogging and weight lifting to all employees, and require employees who are out of shape to participate.

Which of the following, if true, provides the most support for the personnel officer's proposal?

(A) The medical expenses incurred by fit people who participate in a program of jogging and weight lifting are less than those incurred by fit people who do not participate in such a program.

(B) More otherwise fit people are injured by participating in rigorous jogging and weight-lifting programs than are injured by participating in moderate jogging and weight-lifting programs.

(C) The likelihood of incurring medical expenses is slightly greater for people who participate in fitness programs offered by their employers than it is for people who participate in programs offered commercially.

(D) Moderate fitness programs increase the average person's fitness to the same extent that rigorous fitness programs do.

(E) More health problems and injuries are the result of mandatory participation in a fitness program than are the result of voluntary participation.
 

Front

 

Conclusion: Offer a rigorous fitness program tới tất cả employees + YÊU CẦU employees who are out of shape THAM GIA => SẼ giảm được health-insurance costs

WHY: Nhân viên out of shape thì có viện phí cao hơn fit employees

Prethink + Strengthen: Phải liên quan đến program này và viện phí

 

A. Viện phí mà những fit employees có tham gia chương trình này THẤP HƠN là những fit employees KO tham gia chương trình.

=> Ko chỉ chú tâm vào employees who are out of shape, cả những  fit employees cũng phải giảm viện phí, thay vì tăng, thì plan mới hiệu quả được

D. Việc tăng fitness không liên quan gì đến viện phí

Back

 

Front

 

Conclusion: Heat water, tiết kiện điện, bảo vệ vải, không làm nóng hairpins là KO ĐỦ

Strengthen:

E. Tác giả chưa nghĩ đến trường hợp là cái máy có làm nóng thick metal parts hay ko

Back

Researchers took a group of teenagers who had never smoked and for one year tracked whether they took up smoking and how their mental health changed. Those who began smoking within a month of the study's start were four times as likely to be depressed at the study's end than those who did not begin smoking. Since nicotine in cigarettes changes brain chemistry, perhaps thereby affecting mood, it is likely that smoking contributes to depression in teenagers.

Which of the following, if true, most strengthens the argument?


(A) Participants who were depressed at the study's start were no more likely to be smokers at the study's end than those who were not depressed.

(B) Participants who began smoking within a month of the study's start were no more likely than those who began midway through to have quit smoking by the study's end.

(C) Few, if any, of the participants in the study were friends or relatives of other participants.

(D) Some participants entered and emerged from a period of depression within the year of the study.

(E) The researchers did not track use of alcohol by the teenagers.

Front

Conclusion: Smoking dẫn tới depression

WHY: Vì những người bắt đầu smoke khi study bắt đầu thì có khả năng depressed gấp 4 lần so với những người không smoke

A=> Cho dù Depression một thời gian cũng ko dẫn tới smoking

=> Những người bị Depression trước khi study's start sẽ không hút thuốc khi bắt đầu study

-> Strengthen

 

Back
Front

Conclusion: Vaccine là nguồn của con virus TÌM THẤY Ở MESOTHELIOMAS các thập kỉ sau đó

WHY:

- 60% số người có tế bào bị mesotheliomas thì có chứa SV40

- SV40 là nguyên nhân đóng góp lên MESOTHELIOMAS

- in 1960 some polio vaccine was contaminated with the virus.

 

C. Đã cho trong bài " in 1960 some polio vaccine was contaminated with the virus."

  • Không có nhắc gì đến MESOTHELIOMAS

E. Vaccine KO nhiễm vius -> MESOTHELIOMAS ko chứa SV40

Back

 

Front

 

Conclusion: Việc chuyển từ canola oil sang coconut oil không làm tổn hại đến popcorn sales đâu

Why: Vì sales của popcorn tăng 5%

Strengthen:

A. Popcorn mới outperform other refreshments => strengthen

B. Lợi nhuận từ Food and beverage sales > lợi nhuận từ sales of movie tickets.

-> không liên quan

 

C. Customers thích popcorn popped in coconut oil

D. Total attendance tăng 20%, trong khi popcorn chỉ tăng 5%

E. Năm trước nữa tăng 10%, trong khi năm ngoái chỉ tăng 5%

-> weaken quan điểm của critic, ủng hộ Moviemania

Back

If new working practices raise a firm's productivity, will the firm respond by paying its workers more? Not in a competitive market. In such a market the firm, to gain a competitive edge, will reduce prices. The workers' real wages, as measured by those wages' purchasing power, will still rise because of lower prices.

In a competitive market which of the following, if true, ensures that the workers of a firm that achieved productivity gains will derive from these gains the benefit of higher real wages?

(A) The workers' firm continues to achieve productivity gains.
(B) Other firms do not achieve comparable productivity gains.
(C) The workers buy products made by the firm that employs them.
(D) The workers prefer the new working practices over the old.
(E) The firm pays its workers at or above the industry's average.

Front

1. New working practices -> tăng productivity

2. Công ty không tăng lương, chỉ giảm giá sản phẩm

3. Lương THỰC TẾ của nhân viên sẽ tăng: bởi vì giá sản phẩm giảm-> sức mua cao hơn

 

Điều gì đảm bảo nhân viên sẽ hưởng lợi từ "Lương thực tế tăng"?

A. Công ty tiếp tục tăng productivity -> ko có ích gì! Vốn dĩ productivity vẫn đang tăng

B. CÔNG TY KHÁC ko đạt được comparable productivity gains: Mức tăng năng suất NGANG BẰNG VỚI CÔNG TY NÀY

-> sản phẩm của cty khác ko giảm giá -> Sức mua giữ nguyên, đâu có hưởng lợi gì

D. Workers có thích new working practices hay ko? ĐÂU CÓ AI quan tâm đến sở thích của nhân viên. Miễn productivity tăng là được

E. Lương hiện tại có cao hơn trung bình của thị trường hay không? -> đâu hưởng lợi gì.

Công ty đâu có tăng lương đâu

C. Nhân viên mua hàng công ty mình- giá sản phẩm đã giảm -> được hưởng lợi

Back

Assumption

(20 cards)

Front

Conclusion: Giá sau thuế -> giảm doanh thu của thuốc lá

Why: Vì năm ngoái tăng 8 cent thuế -> sales giảm 10%

Năm trước đó, sales chỉ giảm 1%

 

Gap: tăng 8 cent thuế -> giá sau thuế sau đó tăng/ giảm?

Negate assumption: BREAK ARGUMENT - Giá sau thuế ko liên quan gì đến doanh thu đâu

A. Negate -> Giá trước thuế tăng= lượng tăng thuế-> giá sau thuế vẫn tăng tương quan với doanh thu giảm

-> ko break được argument

B. is not required for drawing the conclusion.

C. is not required for drawing the conclusion.

 

D. Negate -> giá trước thuế giảm nhiều hơn 8 cent

=> giá sau thuế giảm

Trong khi đó doanh thu giảm

=> giá sau thuế ko có tương quan MẠNH đến doanh thu

 

E. is not required for drawing the conclusion.

Back
Front

Conclusion: Mosaics (tranh ghép khảm) ở Sep rất có khả năng là được tạo ra từ những nghệ nhân du lịch đến từ vùng khác của Roman

Why: Vì những Mosaics này miêu tả những con vật mà không sống ở Sep vào thời kì này.

Neagte Assumption -> BREAK argument

A. B. Stones/ single, two, hay three-> ko quan tâm

C. Lặp lại đề bài

D. Lặp lại đề bài

E. Negate= có một cái common repertory of  mosaic designs  mà tất cả các artisans đều quen thuộc -> Cái kho tài liệu này có thông tin về các loài vật kia

->Những nghệ nhân ở Sep cũng có thể biết và tạo ra những Mosaics này

 

 

Back
Front

 

Conclusion: Long-term gains từ những tickets mua trước 2 ngày sẽ RẤT LỚN

WHY: Những ticket này không được refund nếu không sử dụng

Negate đáp án = break conclusion= Long-term gains sẽ KHÔNG LỚN

 

A. More discounted, advance purchase tickets are purchased than are actually used => có GAINS từ những vé mua nhưng ko sử dụng

-> Negate: Không có discounted, advance purchase tickets không được sử dụng

=> Không có thêm revenue nào so với trước kia cả

 

D. Có discontinue 30day advance purchase tickets hay không cũng chả ảnh hưởng gì tới revenue

E. Bán cho người chưa từng bay/ đã từng bay -> Ko liên quan. Quan trong là tổng số vé bay trong thời điểm này có tăng hay không? Revenue thu được như thế nào? Chi phí bỏ ra như thế nào

Back

In Brindon County, virtually all of the fasteners—such as nuts, bolts, and screws—used by workshops and manufacturing firms have for several years been supplied by the Brindon Bolt Barn, a specialist wholesaler. In recent months many of Brindon County’s workshops and manufacturing firms have closed down, and no new ones have opened. Therefore, the Brindon Bolt Barn will undoubtedly show a sharp decline in sales volume and revenue for this year as compared to last year.

The argument depends on assuming which of the following?

(A) Last year the Brindon Bolt Barn’s sales volume and revenue were significantly higher than they had been the previous year.

(B) The workshops and manufacturing firms that have remained open have a smaller volume of work to do this year than they did last year.

(C) Soon the Brindon Bolt Barn will no longer be the only significant supplier of fasteners to Brindon County’s workshops.

(D) The Brindon Bolt Barn’s operating expenses have not increased this year.

(E) The Brindon Bolt Barn is not a company that gets the great majority of its business from customers outside Brindon County.

Front

 

Conclusion: Sale của công ty này sẽ giảm MẠNH so với năm ngoái

WHY: Nhiều đối tác mua hàng trong vùng đã đóng cửa và không có mở mới các công ty tương đương

NEGATE ASSUMPTION: -> Đối tác trong vùng đóng cửa sẽ KHÔNG khiến Sales của công ty giảm MẠNH

 

B. Dù các công ty trong vùng KHÔNG giảm volume of work, nếu volume of work BẰNG năm ngoái thì sales của công ty vẫn giảm MẠNH

E. Nếu công ty có PHẦN LỚN biz ngoài vùng này -> Sales KHÔNG giảm MẠNH, có thể giảm nhưng chỉ GIẢM ÍT

Back
Front

Conclusion: Tăng số lượng radio stations lên nhưng số lượng người nhận được special program sẽ không tăng mạnh đâu

Vì sao: Vì số lượng radios không tăng

Assumption: Số lượng radios <tương đương> số lượng người nhận special program sao?

Negate assumption: BREAK conclusion - Số lượng người nhận được special program sẽ tăng mạnh

Negate A. Có nhiều radios stations tiếp cận đến những vùng mà trước đây không phủ sóng, các vùng này trước đây đã có radios rồi

-> Số lượng người nhận được special programs sẽ tăng

B. Tất cả/ không phải tất cả những người số ở trong vùng này đều có radios

D. Người không có radios thì không thể nhận chương trình

E. Station không offer cùng loại chương trình

-> ko ảnh hưởng gì đến đáp án

Negate C. thêm công nghệ RDS sẽ giảm khu vực phát sóng của station -> strengthen

Back
Front
Back

 

Front

Back ground: 90% các dự án bị canceled là của các đảng đối lập

Conclusion: Lựa chọn của President được thúc đẩy bởi chính sách ngân sách hợp lý, không liên quan đến chính trị giữa các đảng

Why: Các dự án bị canceled này được xác định là wasteful trong báo cáo bởi các kiểm toán trung lập chuyên nghiệp

Negate Assumption: Break conclusion: Lựa chọn của President là để punish các districts của các đảng đối lập

Missing link: Còn các dự án không bị cancel thì sao, có cái nào wasteful không

 

Negate A. Canceling highway projects có phải là "cách duy nhất" hay không

Negate E. Reports này có được coi là khách quan hay không khách quan

Negate D. Các dự án được thực hiện bởi đảng của President có more expensive hơn hay không

-> cũng ko thể khẳng định President có mục đích punish các districts của đảng đối lập

 


E. Weaken

 

Negate B. Có nhiều dự án wasteful khác được thực hiện bởi đảng của President

-> chứng tỏ President punish các đảng đối lập

Back

Ramirez: The film industry claims that pirated DVDs, which are usually cheaper than legitimate DVDs and become available well before a film's official DVD release date, adversely affect its bottom line. But the industry should note what the spread of piracy indicates: consumers want lower prices and faster DVD releases. Lowering prices of DVDs and releasing them sooner would mitigate piracy's negative effect on film industry profits.

The argument above relies on which of the following assumptions?

A. Releasing legitimate DVDs earlier would not cause any reduction in the revenue the film industry receives from the films' theatrical release.
B. Some people who would otherwise purchase pirated DVDs would be willing to purchase legitimate DVDs if they were less expensive and released earlier than they are now.
C. The film industry will in the future be able to produce DVDs more cheaply than is currently the case.
D. Some current sellers of pirated DVDs would likely discontinue their businesses if legitimate DVDs were released faster and priced lower.
E. Current purchasers of pirated DVDs are aware that those DVDs are not authorized by the film industry.
 

Front

 

Conclusion: Giảm giá đĩa DVDs và phát hành sớm hơn sẽ giảm được ảnh hưởng tiêu cực của đĩa DVDs lậu (tăng lợi nhuận/ doanh số = bottom line)

WHY: Đĩa DVDs lậu rẻ và phát hành sớm hơn a film's official DVD, mà khách hàng lại muốn vậy

Assumption: Negate đáp án -> CHẮC CHẮN BREAK conclusion: Giảm giá và phát hành sớm CHẮC CHẮN KHÔNG giảm được ảnh hưởng tiêu cực từ đĩa lậu (-> tăng bottomline)

 

A.Negate: Có reduction in revenue from theater. Tuy nhiên ko break argument. Nếu doanh thu tăng từ DVDs đủ bù đắp reduction từ theater thì vẫn tăng bottom line

-> Chỉ weaken argument thôi nhưng không BREAK hẳn
Vẫn chứa 1 khả năng là argument đúng

 

B. Negate: Không ai chuyển từ DVDs lậu sang DVDs bản quyền-> Ko có tăng được bottom line

D. Ko có seller of pirated DVDs nào DISCONTINUE their biz -> ko sao. Dù ko discontinue nhưng doanh số của họ giảm đi là đã giảm được ảnh hưởng tiêu cực từ đĩa lậu 

Back
Front

 

Negate đáp án ->  đưa ra một nguyên nhân khác với đề bài => CHỌN

Back
Front

 

Conclusion: Miễn là công nghệ chống dò dỉ dầu hiệu quả, fear về việc " Xây dựng oil pipeline dưới đáy làm giảm số lượng cá" là groundless= không có căn cứ

Negate Assumption: Vẫn có khả năng "Xây dựng oil pipeline dưới đáy làm giảm số lượng cá"

A. new industrial development -> không liên quan đến việc Xây dựng oil pipeline

B. Không có lý do để tin rằng công nghệ không hiệu quả -> Ko liên quan. Vì conclusion đã bao gồm " miễn là công nghệ hiệu quả"

D. Damage tới số lượng cá là sự thiệt hại duy nhất hay không? ->

Gây hại cho loài khác cũng ko liên quan

E. Các loài cá là giống nhau trước và sau khi hồ bị ô nhiễm -> ko liên quan

C. Negate: Đáy hồ CÓ CHỨA chất độc hại sẽ bị khuấy lên nếu xây dựng pipeline => Số lượng cá có thể sẽ bị giảm

Back

Although the school would receive financial benefits if it had soft drink vending machines in the cafeteria, we should not allow them. Allowing soft drink machines there would not be in our students' interest. If our students start drinking more soft drinks, they will be less healthy.

The argument depends on which of the following?


(A) If the soft drink vending machines were placed in the cafeteria, students would consume more soft drinks as a result.

(B) The amount of soft drinks that most students at the school currently drink is not detrimental to their health.

(C) Students are apt to be healthier if they do not drink soft drinks at all than if they just drink small amounts occasionally.

(D) Students will not simply bring soft drinks from home if the soft drink vending machines are not placed in the cafeteria.

(E) The school's primary concern should be to promote good health among its students.

Front

Conclusion: Không nên cho phép đặt máy ở cafeteria

Why: Học sinh uống nhiều nước ngọt sẽ bị less healthy

Gap: Mang máy đến _____ Học sinh bắt đầu uống nhiều nước ngọt hơn

 

Negate Assumption: Đặt máy ở đó thì học sinh cũng không uống nhiều nước ngọt hơnkhông bị less healthy

 

Neagte A. Có vending machine thì học sinh vẫn CHẮC CHẮN KO UỐNG NHIỀU nước ngọt hơn -> CHẮC CHẮN KO less healthy

B. Hàm lượng nước ngọt đa số học sinh uống ít hay nhiều -> ko quan tâm. Quan trọng là học sinh có bị less healthy hơn không?

Negate D = Học sinh có thể mang nước ngọt từ nhà đi (Supplier 1), nhưng nếu có thêm vending machines (Supplier 2) ở trường thì học sinh có thể mua thêm nếu uống hết nước ngọt ở nhà. -> SỰ TIỆN LỢI từ vending machine khiến học sinh uống nhiều nước ngọt hơn -> vẫn có thể bị less healthy

E. Tồn tại một cái recommendation "Nhà trường SHOULD/ SHOULDN'T promote good health" chả ảnh hưởng gì đến health của học sinh cả

Back
Front

 

Conclusion: vòng tròn (Có người trốn thuế -> nhà nước tăng % thuế -> càng nhiều người trốn thuế -> nhà nước tăng % thuế ) sẽ không xảy ra, trừ khi.....

 

C. Nếu lawmakers đã tính đến việc sẽ có người trốn thuế nên đã tính % thuế đủ để đạt được lượng revenue cần thiết -> ko cần phải tăng % thuế nữa-> ko có cycle này xảy ra

Back

A recent report determined that although only three percent of drivers on Maryland highways equipped their vehicles with radar detectors, thirty-three percent of all vehicles ticketed for exceeding the speed limit were equipped with them. Clearly, drivers who equip their vehicles with radar detectors are more likely to exceed the speed limit regularly than are drivers who do not.

The conclusion drawn above depends on which of the following assumptions?


(A) Drivers who equip their vehicles with radar detectors are less likely to be ticketed for exceeding the speed limit than are drivers who do not.

(B) Drivers who are ticketed for exceeding the speed limit are more likely to exceed the speed limit regularly than are drivers who are not ticketed.

(C) The number of vehicles that were ticketed for exceeding the speed limit was greater than the number of vehicles that were equipped with radar detectors.

(D) Many of the vehicles that were ticketed for exceeding the speed limit were ticketed more than once in the time period covered by the report.

(E) Drivers on Maryland highways exceeded the speed limit more often than did drivers on other state highways not covered in the report.

Front

1. Conclusion: drivers who equip their vehicles with radar detectors are more likely to exceed the speed limit regularly than are drivers who do not.
<=> Bất phương trình: Số lượt exceed thực tế_xe có radar detectors/ tổng số xe radar > Số lượt exceed thực tế_xe 0 radar/ tổng số xe 0 radar
*Mình ghi chữ thực tế ở đây vì: có thể có những xe_exceed the speed limit nhưng không bị ticketed

2. WHY: số xe có radar detectors/ tổng số xe = 3%, số xe có radar detectors_ticketed/ tổng số xe_ticketed = 30%

3. Với những bài assumption, bạn có thể nhanh chóng nhìn ra gap nếu để ý kết nối cái WHY với cái Conclusion
WHY=> xe có radar detectors are more likely to be ticketed
Nhưng conclusion=> xe có radar detectors are more likely to exceed speed limit
=> Gap: bị ticketed # với exceed speed limit

 

(A) Drivers who equip their vehicles with radar detectors are less likely to be ticketed for exceeding the speed limit than are drivers who do not.
=> NẾU như (A) - xe có radar detectors còn ít khả năng bị ticketed hơn THÌ thực tế là xe có radar detectors còn exceed speed limit nhiều hơn là tác giả đang nghĩ
=> strengthen conclusion.
Nhưng ở đây mình không cần một câu strengthen, mình đang cần tìm tác giả đang có assumption gì (gap trong lập luận của tác giả là gì)

(B) Drivers who are ticketed for exceeding the speed limit are more likely to exceed the speed limit regularly than are drivers who are not ticketed.
-> chính là cây cầu bắc qua cái gap bên trên.
(Premise) xe có radar are more likely to be ticketed + (Bridge) xe mà bị ticketed are more likely exceed speed limit -> (Conclusion) xe có radar are more likely exceed speed limit
Nếu negate câu B: xe mà bị ticketed are LESS likely exceed speed limit => Conclusion phải là: xe có radar are LESS likely exceed speed limit
=> Break conclusion ban đầu

Back
Front

 

Conclusion: Kế hoạch chống làm giả sẽ trở lên thành công

Why: Ngân hàng đã phát hành ra các ngân phiếu có những cái chấm QUÁ nhỏ -> KHÓ mà có thể dupplicated một cách chính xác bởi các máy scanner currently available

- Khi mà những ngân phiếu này được scanned và printed, những chấm nhỏ này sẽ trộn với nhau tạo thành chữ "VOID" trên ngân phiếu

A.B.C.D. Tỷ lệ counterfeit checks/ Giá trị của những cái check/ Khả năng nhận biết những cái dots  bằng mắt/ Số tiền các tập đoàn phải trả cho những cái new checks

-> ko liên quan

E. Negate. Nếu những máy scanner trong tương lai gần có thể in lại một cách xác những cái dots nhỏ hơn (ví dụ = những cái dot của ngân hàng) thì những cái dots này sẽ không bị blend -> Kế hoạch chống làm giả ko thành công

Back
Front

Conclusion: Sẽ giảm được chi phí bảo hiểm - sức khoẻ NẾU bắt buộc toàn bộ nhân viên tham gia chương trình fitness

WHY: Tất cả employees who are fit có lower doctor bills và shorter hospital stays SO VỚI employees who are out of shape

ASSUMPTION: Negate đáp án -> việc bắt toàn bộ nhân viên tham gia chương trình fitness sẽ KHÔNG giảm được chi phí bảo hiểm-sức khoẻ

 

B. Những người BỊ YÊU CẦU tham gia fitness program (những người out of shape) (sau khi tham gia) thì SẼ có chi phí y tế thấp hơn những người KHÔNG BỊ YÊU CẦU THAM GIA (những người đã fit)

=> Negate. Những người BỊ YÊU CẦU tham gia fitness program (những người out of shape) (sau khi tham gia) thì SẼ KHÔNG có chi phí y tế thấp hơn những người KHÔNG BỊ YÊU CẦU THAM GIA (những người đã fit). 

Nhưng những người out of shape chỉ cần giảm 1 ít chi phí là đã giảm được tổng chung rồi

=> Ko break reasoning

 

C. Negate: Các hoạt động hăng hái mà fitness program yêu cầu TỰ tạo ra các medical expense > chi phí y tế giảm được do chương trình.

-> Side effect > lợi ích

-> Break conclusion

Back

Market researchers recently reported that ninety percent of the people interviewed objected to a particular detergent's advertisement because of their portrayal of women. Yet this detergent is purchased by twenty percent of consumers. So its advertisements must be considered to be unobjectionable to at least twenty percent of consumers.

The conclusion of the argument above depends on which of the following assumption?

(A) People who object to a product's advertisements may still buy that product.

(B) The people who buy this detergent are familiar with its advertisement.

(C) Most of the people who do not buy this detergent consider this advertisements to be objectionable.

(D) Most people wash their own clothes, so they have to buy some brand of detergent.

(E) Most of the people in the market research study were women.

Front

 

Conclusion: Quảng cáo này không bị phản đối = unobjectionable bởi ít nhất 20% TẤT CẢ customers

Why:

- Có 20% người mua hàng

- Tuy rằng có 90% người phỏng vấn phản đối

 

Gap: Mua hàng # không phản đối ads

Negate Assumption: Việc có 20% người mua hàng - không thể kết luận có ít nhất 20% TẤT CẢ customers không phản đối

 

A. Weaken-> Nếu người phản đối mà vẫn mua hàng -> khả năng có ít hơn 20% người không đối

C. Weaken. Người không mua hàng thì phản đối -> Không thể có nhiều hơn 20% người không phản đối

D.E. Không liên quan

B. ≈ Gap. Nếu những người mua hàng không quen với cái ads thì không thể kết luận mua hàng = không phản đối ads 

Back

Newspaper editorial: In an attempt to reduce the crime rate, the governor is getting tough on criminals and making prison conditions harsher. Part of this effort has been to deny inmates the access they formerly had to college-level courses. However, this action is clearly counter to the governor’s ultimate goal, since after being released from prison, inmates who had taken such courses committed far fewer crimes overall than other inmates.

Which of the following is an assumption on which the argument depends?

(A) Not being able to take college-level courses while in prison is unlikely to deter anyone from a crime that he or she might otherwise have committed.

(B) Former inmates are no more likely to commit crimes than are members of the general population.

(C) The group of inmates who chose to take college-level courses were not already less likely than other inmates to commit crimes after being released.

(D) Taking high school level courses in prison has less effect on an inmate’s subsequent behavior than taking college-level courses does.

(E) The governor’s ultimate goal actually is to gain popularity by convincing people that something effective is being done about crime.

Front

Conclusion: Cấm người tù tham gia vào các khoá học cấp cao đẳng sẽ counter to the governor’s ultimate goal (<=> KHÔNG GIẢM ĐƯỢC số lượng crimes)

Why: Sau khi được thả khỏi tù, những người đã từng tham gia các khọc này phạm tội ÍT HƠN so với những người tù khác (Premise 1)

Assumption: Negate đáp án -> BREAK argument: Cấm người tù tham gia vào các khoá học cấp cao đẳng SẼ CHẮC CHẮN GIẢM số lượng crimes

 

A. Negate đáp án: Việc KHÔNG được tham gia vào các khoá học này CÓ KHẢ NĂNG ngăn cản một người khỏi phạm tội-> TÍCH CỰC của việc không được tham gia khoá học

-> Tuy nhiên là ĐƯỢC tham gia CŨNG có khả năng GIẢM phạm tội 

=> Ko biết việc cho phép/ không cho phép tham gia thì tốt hơn: Cấm người tù tham gia khoá học thì VẪN CÓ KHẢ NĂNG là số lượng tội phạm được giảm hoặc tăng

->  Không break được conclusion

 

C. Negate đáp án: Những người CHỌN tham gia khoá học BẢN CHẤT LÀ ĐÃ (were ALREADY) ít khả năng phạm tội hơn sau khi ra khỏi tù

-> Việc không có khoá học sẽ khiến việc ngồi tù harsher và giảm crime rate

-> Break cái reasoning của tác giả.

* Nguyên nhân của hiện tượng "những người chọn tham gia khoá học phạm tội ít hơn sau khi ra tù" KHÔNG PHẢI vì khoá học, MÀ LÀ VÌ bản chất họ đã thế.

Back

In a study conducted in Pennsylvania, servers in various restaurants wrote “Thank you” on randomly selected bills before presenting the bills to their customers. Tips on these bills were an average of three percentage points higher than tips on bills without the message. Therefore, if servers in Pennsylvania regularly wrote “Thank you” on restaurant bills, their average income from tips would be significantly higher than it otherwise would have been.

Which of the following is an assumption on which the argument relies?


(A) The “Thank you” messages would have the same impact on regular patrons of a restaurant as they would on occasional patrons of the same restaurant

(B) Regularly seeing “Thank you” written on their bills would not lead restaurant patrons to revert to their earlier tipping habits

(C) The written “Thank you” reminds restaurant patrons that tips constitute a significant part of the income of many food servers

(D) The rate at which people tip food servers in Pennsylvania does not vary with how expensive a restaurant is

(E) Virtually all patrons of the Pennsylvania restaurants in the study who were given a bill with “Thank you” written on it left a larger tip than they otherwise would have.

Front

 

Conclusion: Viết THANK YOU vào hoá đơn REGULARLY thì trung bình tiền tips nhận được cao hơn RẤT NHIỀU so với trường hợp KHÔNG viết THANK YOU

Why: Viết THANK YOU  vào một số hoá đơn ĐÃ ĐƯỢC CHỌN RANDOMLY thì tiền tips từ những hoá đơn này cao hơn 3% so với những hoá đơn không viết

Negate Assumption: Dù viết THANK YOU thì tiền tips cũng KHÔNG TĂNG MẠNH

 

A. So sánh regular patrons vs. occasional patrons-> irrelevant

C. remind -> irrelevant

D. rate có vary theo mức độ nhà hàng hay không -> irrelevant

E. Negate: KHÔNG PHẢI TOÀN BỘ patrons trong STUDY nhận được bill có chứa thanhk you đều đưa larger tip

-> Cũng ko sao. ĐA SỐ MỌI NGƯỜI đưa tips cao hơn là đủ để average income tăng MẠNH RỒI

B. Negate: Nhìn thấy thank you "REGULARLY" khiến người ta quay trở về thói quen tips cũ

->Break

Back
Front

Conclusion: Rancher sẽ tiếp tục kinh doanh tốt trong tương lai mà không sợ nguy hiểm về Disease X

WHY: Dùng phương pháp trị liệu gene sẽ nhỏ tận gốc (eradicate) cái genetic cause - đã được phát hiện- of the disease.

 

Prethink: Negate đáp án -> BREAK ARGUMENT

-> Dù gene therapy có trị tận gốc genetic cause thì Rancher vẫn tiếp tục nguy hiểm về Disease X

-> Cần phải speak out cái broken conclusion này ra

 

A. The gene therapy will not cause any defects in cattle.

-> Không liên quan gì đến Disease X-> sai

-> Negate: Phương pháp trị liệu gene có thể gây ra một số khuyết điểm lên gia súc-> thì sao? Disease X vẫn bị tiêu diệt mà

 

D. Có disease nào đó khác đang threaten lên cattle

-> Không liên quan gì đến Disease X -> sai

 

E. There are no precursors to or triggers of Disease X, genetic or otherwise, other than the one targeted by the new gene therapy

-> Không có hạt nhân/ triggers nào KHÁC của Disease X, ngoài cái bị targeted by the new gene therapy

-> Negate: Có những nguyên nhân khác dẫn đến Disease X chưa bị target

-> Break conc

Back
Front

 

* Assumption: không được trái ngược lại với thông tin trong bài

 

Conclusion: Newly thin person sẽ tăng cân trở lại cho đến khi body size match their metabolic rate.

Why: Vì metabolism của newly thin person không thay đổi -> họ sẽ burn ít calo ở mức cân mới hơn so với những người bình thường đã ở mức cân đó

Negate Assumption: BREAK conclusion -  newly thin person sẽ không tăng cân trở lại dù burn ít calo hơn đâu

 

Negate A. Newly thin person sẽ ăn ÍT hơn RẤT NHIỀU so với người bình thường đã ở mức cân đó -> họ sẽ consume ít hơn RẤT NHIỀU calo và không tăng cân trở lại

 

C. Trái ngược với thông tin trong bài.

Bài nói newly thin person sẽ có metabloism rate thấp và burn ít calo hơn. -> amount of calo BURNED phụ thuộc vào weight và không liên quan gì đến lượng calo CONSUMED

  • Mà determined là như thế nào? Tốc độ burn giảm khi giảm food có nhanh hơn hay chậm hơn tốc độ bủn giảm khi giảm cân? Việc này có ảnh hưởng gì đến việc tăng cân trở lại??

 

 

Back

EVALUATION

(5 cards)

Front

 

B. % export dù có nhiều, nhưng tổng lượng ít thì vẫn export ít

E. Nếu sau khi realign (tổ chức lại), maximum capacity tăng so với current capacity thì strengthen

Ngược lại sẽ là weaken

Back
Front

 

Nếu sản phẩm đã bị hỏng từ trước thì có packing kiểu gì cũng sẽ phải return

Back
Front

 

Conclusion: magnetic fields from high-voltage power lines KHÔNG CÓ KHẢ NĂNG gây ra vấn đề sức khoẻ

WHY: Beyond a distance of a few feet, magnetic fields from high-voltage power lines YẾU HƠN so với the average strength of magnetic fields in homes that are not located near such lines.

Prethink: magnetic fields from high-voltage power lines VẪN CÓ KHẢ NĂNG gây ra vấn đề sức khoẻ

 

C. Dù high-voltage power lines are routed near residential dwellings in urban areas

Kể cả ở gần thì argument cũng ko có nói là có health problems hay không

=> Không đánh giá được

A. Nếu magnetic fields in homes có thể gây vấn đề sức khoẻ thì vẫn có khả năng magnetic fields from high-voltage power lines (dù yếu hơn) có thể gây vấn đề sức khoẻ

Back
Front

Conclusion: Magazine Publisher (MP) không theo tư tưởng liberal,

WHY: Đúng là 1 review ban đầu mang quan điểm conservative (bảo thủ) + họ đã ko publish trước khi mời 1 người quan điểm liberal review,  nhưng họ chỉ muốn thể hiện đa dạng các quan điểm nên mới

B. Nếu review đầu mang quan điểm liberal-> thì họ có kiếm những review khác (vd. quan điểm conservative) hay không?

D. Cuốn sách mang quan điểm gì không liên quan

-> vì bọn MP chỉ review chứ không viết sách

Back
Front

 

Dù bác sĩ không muốn nhưng bệnh nhân gây áp lực phải kê đơn những loại thuốc không hợp lý -> quảng cáo vẫn có hại

Back

Paradox

(12 cards)

Front

Fact 1: Bệnh nhân mà mất ngủ thì thấy không bị trầm cảm

Fact 2: Sleep deprivation (loại bỏ giấc ngủ) không được sử dụng như là 1 phương pháp trị liệu mặc dù phương pháp truyền thống có nhiều serious side effects

Prethink: Không dùng sleep deprivation đâu, vì: 

- Sleep deprivation có serious side effects gì?

- Sleep deprivation không có tác dụng lâu dài

- Phương pháp truyền thống có advantages gì?

<=> Cần so sánh Sleep deprivation vs. Conventional treatments

 

A. missing a night's sleep induces a temporary sense of euphoria (trạng thái phấn khích)

C. Mất ngủ lâu dài có thể dẫn dẫn đến suy yếu tạm thời

-> dù là gây trạng thái gì cũng chỉ là temporaryvà không so sánh được với Conventional treatments

D. Không trace được sự thay đổi hoá học trong não bộ

B. Giữ depressed patients awake có tác dụng gì? Mình không được assume

 

E. Depression sẽ trở lại ngay khi bệnh nhân ngủ dù 1 vài phút

=> Sleep deprivation không có tác dụng lâu dài

Back
Front

Fact 1: Bộ y tế yêu cầu không tăng giá thuốc kê đơn, nhằm mục đích giảm chi phí thuốc trên đầu người

Fact 2: Lượng sử dụng thuốc kê đơn không tăng, nhưng chi phí thuốc kê đơn trên đầu người tăng

 

Tại sao?

 

E. Một số hãng nước ngoài dừng bán ở Voronia-> Người dân có thể mua sản phẩm trong nước hoặc của ngước khác.

Nhưng mà mình đâu biết các hãng mới này giá cao hơn hay thấp hơn

-> Ko giải thích được tại sao chi phí trên đầu người vẫn tăng

 

A. Sau khi việc tăng thuốc bị cấm, các hãng thuốc tập trung sản xuất sản phẩm mới để thay thế sản phẩm cũ

-> CÓ THỂ là sản phẩm mới giá cao hơn sản phẩm cũ

=> Người dân mua sản phẩm mới sẽ có chi phí trên đầu người cao hơn

 

Back
Front

Gecko: Con tắc kè

congregate: tụ họp

 

Fact 1: Ở thành thị, house gecko đang displacing the previously established mourning gecko

Fact 2: Ở nông thôn xa khu vực của con người, dân số của 2 loài này ổn định hơn

Fact 3: House gecko không tấn công mourning gecko, nhưng ở nơi mà có nhiều insects tụ họp thì nó ngăn cản mourning gecko ăn insects

=> Prethink: House gecko ở thành thị áp đảo là vì? Có gì liên quan đến INSECTS đã khiến house gecko áp đảo ở thành thị?

 

A. geckos NÓI CHUNG được value

B. geckos bảo vệ lãnh thổ khỏi same species

C. house geckos được đưa đến islands này bằng cách nào

E. geckos sinh sản vô tính/ hữu tính

=> không giải thích được tại sao có sự chênh lệch dân số

 

D. Nơi có các toà nhà sáng (Thành thị) -> insects tập trung quanh ánh sáng

=> Đây là nơi house gecko áp đảo -> số lượng house gecko ở thành thị nhiều hơn

Back
Front

Fact 1: Hãng bảo revenue của họ bị giảm

Fact 2: Retailers bảo mặc dù revenue của họ giảm nhưng revenue của riêng hãng này tăng, nhờ sản phẩm tiên tiến mới

Tại sao revenue của hãng giảm?

 

E. Retailers bán những sản phẩm cũ còn sót lại với giá thấp hơn nhiều

-> Cũng chỉ bán 1 số sản phẩm cũ thôi.

Còn sales của hãng đến từ những sản phẩm mới vẫn tăng mà.

Vẫn ko giải thích được tại sao sales của hãng giảm

 

C. Một lượng lớn revenue của hãng đến từ việc chế tạo components cho other consumer-electronics manufacturers

-> hãng bị giảm revenue là từ đây

Back
Front

Fact 1: Dân số ở Mega tăng với tốc độ khá đều trong một thập kỉ qua

Fact 2: 100 năm trước, poor sanitation dẫn đến tỉ lệ tử vong cao và cái kích thích dân số là nhập cư từ nông thôn là nhập cư

Fact 3: Nhập cư vẫn tiếp tục và thậm chí tăng, sanitation cũng improve

Fact 4: Nhưng dân số của thành phố không tăng mạnh

 

Prethink - Dân số không tăng mạnh, là vì? Có gì đã kìm hãm tỉ lệ tăng dân số?

 

A. Tỷ lệ tử vong vì poor sanitation thì LỚN HƠN tỉ lệ tử vong vì giao thông (major cause of death hiện tại)

<=> Tỷ lệ tử vong giảm đi do có improved sanitation phải lớn hơn tỉ lệ tử vong tăng do giao thông

=> Đáng nhẽ dân số phải tăng mạnh chứ

 

C. Cities khác/ D. Phần lớn dân nhập cư đến Mega/ E. Mega có better employment

Ko giải thích được tại sao dân số không tăng mạnh

 

B. Tỷ lệ sinh giảm đều -> Giải thích tại sao dân số không tăng mạnh

Back

In 1960, 10 percent of every dollar paid in automobile insurance premiums went to pay costs arising from injuries incurred in car accidents. In 1990, 50 percent of every dollar paid in automobile insurance premiums went toward such costs, despite the fact that cars were much safer in 1990 than in 1960.

Which of the following, if true, best explains the discrepancy outlined above?

(A) There were fewer accidents in 1990 than in 1960.
(B) On average, people drove more slowly in 1990 than in 1960.
(C) Cars grew increasingly more expensive to repair over the period in question.
(D) The price of insurance increased more rapidly than the rate of inflation between 1960 and 1990.
(E) Health-care costs rose sharply between 1960 and 1990.

Front

 

injuries: tổn thương thân thể

 

=> Cars grew increasingly more expensive to repair: ko liên quan đến chi phí bảo hiểm trả cho injuries

Health-care cost thì liên quan

 

* Cần xem xét kĩ nếu thấy 2 contenders

Back

In some countries, real estate agents negotiating with prospective clients who want to sell their houses all recommend initial asking prices that are as high as the current market permits. They do this because potential sellers are more likely to entrust their houses to the agent who values them highest. In a housing market with falling demand, the predictable result is a sharper drop in the number of houses sold than the drop in demand alone would account for.

Which of the following, if true, best explains the result described as predictable above?

(A) A drop in demand for houses depresses prices disproportionately if earlier strong demand stimulated much new construction.

(B) Real estate agents do not earn the commission on houses they help sell until the sale is actually executed.

(C) Many people sell their old house below the asking price in order to obtain the funds to buy a new one

(D) In a housing market with rising demand, even houses that were originally overpriced usually sell at the asking price.

(E) For many sellers of houses, the initial asking price comes to represent the true value, below which they will not sell.

Front

Info:

1. Sellers chọn real estate agents - người mà recommend initial asking prices cao nhất có thể

2. Thị trường có faling demand sẽ có số lượng nhà bán được - giảm nhiều hơn - sự giảm mà demand chịu trách nhiệm

 

Tại sao?

A. Không có thông tin về earlier strong demand

B. Real estate agents earn the commission như thế nào? -> irrelevant

C. Người bán nhà dưới mức asking price # nhóm người mà đang bàn đến (muốn bán nhà ở mức asking price)

Người ta muốn có tiền để mua 1 cái nhà mới -> irrelevant

D. Thị trường với rising demand -> irrelevant

E. Người ta sẽ không bán nhà dưới initial asking price - mức quá cao mà agent recommend -> thêm một nguyên nhân khiến không bán được nhà

-> số lượng nhà bán được giảm  

Back
Front

Fact 1: Bệnh nhân mà mất ngủ thì thấy không bị trầm cảm

Fact 2: Sleep deprivation (loại bỏ giấc ngủ) không được sử dụng như là 1 phương pháp trị liệu mặc dù phương pháp truyền thống có nhiều serious side effects

Tại sao? Prethink: Sleep deprivation có serious side effects gì?/ Sleep deprivation không có tác dụng lâu dài/ Phương pháp truyền thống có advantages gì?

<=> Cần so sánh Sleep deprivation vs. Conventional treatments

A. missing a night's sleep induces a temporary sense of euphoria (trạng thái phấn khích)

C. Mất ngủ lâu dài có thể dẫn dẫn đến suy yếu tạm thời

D. Không trace được sự thay đổi hoá học trong não bộ

-> không so sánh được 2 phương pháp trị liệu depression

 

B. Giữ depressed patients awake có tác dụng gì? 

-> Cũng không so sánh được 2 phương pháp trị liệu depression

Mình cũng không được assume

 

E. Tác dụng trị liệu depression của Sleep deprivation MẤT NGAY SAU KHI ĐI NGỦ

-> Không thể dùng Sleep deprivation làm phương pháp trị liệu depression

Back
Front

 

Fact 1: Ở thành thị, house gecko đang displacing the previously established mourning gecko

Fact 2: Ở nông thôn xa khu vực của con người, dân số của 2 loài này ổn định hơn

Fact 3: House gecko không tấn công mourning gecko, nhưng ở nơi mà có nhiều insects tụ họp thì nó ngăn cản mourning gecko ăn insects

=> Tại sao có sự khác biệt về dân số của gecko ở thành thị và nông thôn?

 

A. geckos nói chung được value

B. geckos bảo vệ lãnh thổ khỏi same species

C. house geckos được đưa đến islands này bằng cách nào

E. geckos sinh sản vô tính/ hữu tính

=> không giải thích được tại sao có sự chênh lệch dân số

D. Nơi có các toà nhà sáng (Thành thị) -> insects tập trung quanh ánh sáng

=> Đây là nơi house gecko áp đảo -> số lượng house gecko ở thành thị nhiều hơn

Back

Archaeologists have found wheeled ceramic toys made by the Toltec, twelfth-century inhabitants of what is now Veracruz. Although there is no archaeological evidence that the Toltec used wheels for anything but toys, some anthropologists hypothesize that wheeled utility vehicles were used to carry materials needed for the monumental structures the Toltec produced.

Which of the following, if true, would most help the anthropologists explain the lack of evidence noted above?


(A) The Toltec sometimes incorporated into their toys representations of utensils or other devices that served some practical purpose.

(B) Any wheeled utility vehicles used by the Toltec could have been made entirely of wood, and unlike ceramic, wood decays rapidly in the humid climate of Veracruz.

(C) Carvings in monument walls suggest that the Toltec's wheeled ceramic toys sometimes had ritual uses in addition to being used by both children and adults as decorations and playthings.

(D) Wheeled utility vehicles were used during the twelfth century in many areas of the world, but during this time wheeled toys were not very common in areas outside Veracruz.

(E) Some of the wheeled ceramic toys were found near the remains of monumental structures.

Front

 

Đọc kĩ câu hỏi, literally!!!

 

Hypothesis: Xe có bánh đã được sử dụng để carry materials needed for the monumental structures the Toltec produced.

Nhưng tại sao anthropologists không tìm thấy bằng chứng Toltec used wheels for anything but ceramic toys??

A. Toltec kết hợp sự hình ảnh của utensils or other devices vào trong đồ chơi

C. Tranh khắc có suggest về việc "wheeled ceramic toys sometimes had ritual uses"

D. wheeled toys were not very common in areas outside Veracruz.

E. wheeled ceramic toys were found near the remains of monumental structures.

-> ko giải thích được tại sao

 

B. wheeled utility vehicles used by the Toltec làm từ gỗ, dễ bị mục nát với thời tiết ẩm ướt -> không còn tồn tại đến hiện nay để làm bằng chứng

Back
Front

 

Fact 1: Dân số ở Mega tăng với tốc độ khá đều trong một thập kỉ qua

Fact 2: 100 năm trước, poor sanitation dẫn đến tỉ lệ tử vong cao và cái kích thích dân số là nhập cư từ nông thôn là nhập cư

Fact 3: Nhập cư vẫn tiếp tục và thậm chí tăng, sanitation cũng improve

Fact 4: Nhưng dân số của thành phố không tăng mạnh

Tại sao?

A. Tỷ lệ tử vong vì poor sanitation thì lớn hơn tỉ lệ tử vong vì giao thông (major cause of death hiện tại)

<=> Tỷ lệ tử vong giảm đi do có improved sanitation phải lớn hơn tỉ lệ tử vong tăng do giao thông

=> Đáng nhẽ dân số phải tăng mạnh chứ

C. Cities khác/ D. Phần lớn dân nhập cư đến Mega/ E. Mega có better employment

Ko giải thích được tại sao dân số không tăng mạnh

 

B. Tỷ lệ sinh giảm đều -> Giải thích tại sao dân số không tăng mạnh

Back
Chú ý
Front

 

 

Note: 

- Cần phải phải xác định keywords/ chủ đề của argument: Maternal

-> đáp án nào không có maternal thì phải loại 

-> KỂ CẢ TRONG TRƯỜNG HỢP KHÔNG BIẾT TỪ MỚI CŨNG CÓ THỂ LOẠI ĐƯỢC NẾU BIẾT CHỦ ĐỀ

(bear: chịu đựng)

 

B. Lượng mùi mà rat pups sản xuất ra tăng lên khi có sự hiện diện của female rat - không chịu chúng

=> Ko liên quan gì đến maternal

 

E. Sự phát triển maternal interest của female rat đối với rat pup bị inhibited bởi mùi của rat pup

=> Nếu bỏ đi sense of smell của female rat hoặc sent-producing gland của rat pup thì sẽ kích thích quá tình phát triển maternal interest của female rat

Back

Complete & Others

(1 card)

Which of the following provides the most logical completion of the argument?

A large number of wempro corporation's department heads will retire this year. The number of employees with the qualifications now required for promotion to department head equals only half the expected vacancies. Wempro is not going to hire department heads from outside the company, have current department heads take over more than one department, or reduce the number of its departments. So some departments will be without department heads next year, since Wempro will not______.

a) reduce the qualifications for promotion to department head
b) reduce the average number of employees per department
c) reduce the responsibilities of each department
d) promote more than one employee from any department to serve as heads of departments
e) promote any current department heads to higher-level managerial positions

Front

Conclusion: 1 số lượng lớn departments sẽ không có departments head năm sau
WHY: Khi mà công ty KHÔNG ____
+ 1 số lượng lớn department heads nghỉ hưu + số lượng người thay thế đủ qualifications chỉ được 1 nửa vị trí trống 
+ công ty sẽ không thuê department heads từ bên ngoài/ có departments heads hiện tại quản lý thêm deparment khác/ giảm số lượng department

Assumption: Không có cách nào khác giúp công ty tăng lượng department heads tuyển nội bộ?

 

Complete argment: Nếu công ty không làm ____ thì sẽ thêm lý do mà công ty sẽ không có đủ department heads
hoặc infer là: nếu công ty làm___ thì có khả năng sẽ đủ deparment heads

 

A. reduce the qualifications for promotion to department head
-> Công ty không giảm qualifications -> Chắc chắn là ko đủ department heads được promoted
D. Kể cả cho phép nhiều hơn 1 nhân viên từ 1 phòng ban lên làm department heads thì vẫn ko đủ số nhân viên đạt yêu cầu mà.

E. promote any current department heads to higher-level managerial positions
-> Công ty không đưa các department heads hiện tại lên vị trí quản lý cao hơn
-> Kể cả đưa department heads lên vị trí quản lý cao hơn thì càng thiếu department heads

Back